Download as pdf or txt
Download as pdf or txt
You are on page 1of 50

.

VISIONIAS
www.visionias.in

Test Booklet Series

TEST BOOKLET

CSAT ABHYAAS TEST – 3 (5184) – 2024


C
Time Allowed: Two Hours Maximum Marks: 200

INSTRUCTIONS
1. IMMEDIATELY AFTER THE COMMENCEMENT OF THE EXAMINATION, YOU SHOULD CHECK THAT THIS
BOOKLET DOES NOT HAVE ANY UNPRINTED OR TORN OR MISSING PAGES OR ITEMS ETC. IF SO, GET IT
REPLACED BY A COMPLETE TEST BOOKLET.

2. ENCODE CLEARLY THE TEST BOOKLET SERIES A, B, C OR D AS THE CASE MAY BE IN THE APPROPRIATE PLACE
IN THE ANSWER SHEET.

3. You have to enter your Roll Number on the Test Booklet in


the Box provided alongside. DO NOT write anything else on
the Test Booklet.

4. This Test Booklet contains 80 items (Questions). Each item is printed in English. Each item comprises four
responses (answers). You will select the response which you want to mark on the Answer Sheet. In case you
feel that there is more than one correct response, mark the response which you consider most appropriate.
In any case, choose ONLY ONE response for each item.

5. You have to mark all your responses ONLY on the separate Answer Sheet provided. See direction in the
answers sheet.

6. All items carry equal marks. Attempt all items. Your total marks will depend only on the number of correct
responses marked by you in the answer sheet. For every incorrect response one-third of the allotted Marks
will be deducted.

7. Before you proceed to mark in the Answer sheet the response to various items in the Test booklet, you have
to fill in some particulars in the answer sheets as per the instruction sent to you with your Admission
Certificate.

8. After you have completed filling in all responses on the answer sheet and the examination has concluded,
you should hand over to Invigilator only the answer sheet. You are permitted to take away with you the Test
Booklet.

9. Sheets for rough work are appended in the Test Booklet at the end.

DO NOT OPEN THIS BOOKLET UNTIL YOU ARE ASKED


Directions for theTO DO SO 2 (two) items:
following
1 www.visionias.in ©Vision IAS
.
1. In a row of boys and girls, Tanu is 6th from the 4. There are eight cities namely Jabalpur,
beginning while Atul is 12th from the
Kolkata, Lucknow, Mumbai, Nagpur,
beginning. Among the girls, Tanu is at 4th
position from the beginning and 6th position Orangabad, Panipat and Qarimnagar. They are
from the end. Among the boys, Atul is at 7th connected by three modes of transport:
position from the beginning and 24th position
from the end. How many children are there in roadways, airways and railways. The routes
the row? which are connected by road are two-ways,
(a) 46
(b) 48 whereas the routes which are connected by air
(c) 39 and rail are one-way. The cities which are
(d) None of these
connected by roadways are Jabalpur and
2. Study the question and the three statements Panipat, Qarimnagar and Panipat, Lucknow
given below.
and Qarimnagar whereas the cities which are

Question: What is the total compound interest connected by airways are Panipat to Kolkata,
earned at the end of 3 years?
Qarimnagar to Mumbai, Nagpur to Mumbai

Statements: and Nagpur to Lucknow. The cities connected


A. Simple interest at the same rate and for the by railways are Kolkata to Qarimnagar,
same number of years is Rs.4500.
B. The rate of interest is 10% per annum. Lucknow to Jabalpur, Panipat to Orangabad
C. Compound interest for 3 years is Rs.465 and Orangabad to Kolkata.
more than the simple interest for the same
period.
Consider the following statements:
Which of the statements given above is/are
1. If Priya wants to go from Orangabad to
necessary to answer the question?
(a) A and B Mumbai, she need not have to go by road.
(b) B and C
2. If Sachin wants to go from Lucknow to
(c) Either (A and B) or (B and C)
(d) The question cannot be answered even if Kolkata by travelling via the longest
we use all the three statements. possible route, he will visit three other

3. A cloth mill pays different daily wages to male cities enroute.


and female workers. If the total daily wage of
15 male and 25 female workers is Rs. 5800
and the total daily wage of 20 male and 30 Which of the above statements is/are correct?
female workers is Rs. 7200, find the wage (a) 1 only
received by 10 females in one day.
(b) 2 only
(a) Rs. 1600
(b) Rs. 1400 (c) Both 1 and 2
(c) Rs. 2500
(d) Neither 1 nor 2
(d) Rs. 1800

www.visionias.in 2 ©Vision IAS


.
5. Consider the question and the two statements 7. The page numbers in a 20 page document are
given below: written as i, ii, iii, …………….. xx. Which of
the following statements is true regarding the
Question: Five software engineers Appu, document?
Bablu, Cibbu, Dabbu and Eva are working on (a) The Roman numeral ‘v’ has been printed
an IT problem. If Eva takes a look at the on eight pages.
problem and then sends it to Bablu, to whom (b) The Roman numeral ‘v’ has been printed
does Bablu send it to? on ten pages.
(c) The Roman numeral ‘x’ has been printed
Statement-1: Appu is a bug specialist and on eleven pages.
hence is the last person to see the problem. (d) None of the above three statements are
Cibbu can take up the problem only after correct in this context.
Bablu.
Statement-2: Bablu or Eva can take up the 8. The cost of two pens, one eraser and three
problem only after Dabbu. sharpeners is rupees 23. The cost of six pens,
three erasers and one sharpener is rupees 45.
Which of the statements given above is/are
The cost of fourteen pens, seven erasers and
needed to answer the question?
twenty-one sharpeners is rupees 161. Find the
(a) Statement-1 alone is sufficient to answer
cost of a pen in rupees.
the question.
(a) 3
(b) Statement-2 alone is sufficient to answer
(b) 4
the question.
(c) 5
(c) Both Statement-1 and Statement-2 are
(d) Cannot be determined
needed to answer the question.
(d) Even Statement-1 and Statement-2
9. In the CBSE class X board exam, 31%
together are not sufficient to answer the
candidates failed in English, 35% failed in
question.
Maths and 22% failed in both the subjects. The
total number of candidates which appeared in
6. 15 years ago the average age of a family of
the exam is 1,66,000. Now, consider the
four members was 40 years. Two children
following statements:
were born in that span of 15 years. The present
1. Number of candidates which failed in at
average age of the family is the same as it was
least one subject is 34%.
15 years ago. If the age of the older child at
present is 8 years more than the younger one, 2. 22,580 candidates passed in English only.
then what is the respective ratio between the
present age of the older child and the present Which of the statements given above is/are
age of the younger child? correct?
(a) 9 : 4 (a) 1 only
(b) 7 : 3 (b) 2 only
(c) 7: 6 (c) Both 1 and 2
(d) 7 : 4 (d) Neither 1 nor 2

www.visionias.in 3 ©Vision IAS


.
10. The combined average marks of students in 13. If Dipti runs at 30 kmph, she reaches her
four batches, namely L1, L2, L3, and L4, of office 17 minutes late, But, when she runs at
Vision IAS coaching center, is 64%. The 40 kmph, she gets late only by 5 minutes.
individual average marks of the L1, L2, L3, What is the time taken by her to cover the
and L4 batches are 60%, 50%, 75%, and 80%, journey in the case wherein she reaches the
respectively. If the average marks of students office on time?
in the L1 and L2 batches together is 54%, and (a) 48 minutes
(b) 31 minutes
that of the L2 and L3 batches together is 64%,
(c) 42 minutes
what is the ratio of the number of students in
(d) 55 minutes
the L4 and L1 batches?
(a) 3 : 2
Directions for the following 5 (five) items:
(b) 2 : 3 Read the following four passages and answer the
(c) 4 : 1 items that follow each passage. Your answers to these
(d) 1 : 4 items should be based on the passages only.
Passage – 1
11. Consider the following figure. Europe is currently in the midst of a heatwave. Italy, in
particular, is expected to face blistering heat, with
temperatures projected to reach 40℃ to 45℃. There’s
even a chance that the current European temperature
record of 48.8℃, set in Sicily in 2021, could be
surpassed. The current heatwave is being caused by an
anticyclone named Cerberus after the three-headed
monster-dog that guards the gates of the underworld in
Greek mythology. It is a high-pressure system, tends
to be slow moving, which is why they persist for days,
How many triangles are there in the figure or even weeks at a time. They often become semi-
permanent features over large areas of land. When
given above?
high-pressure systems form over hot land, in regions
(a) 24
like the Sahara, the stability of the system generates
(b) 26
even hotter temperatures because the already warm air
(c) 21 is heated even more. Research by the
(d) 22 Intergovernmental Panel on Climate Change confirms
this trend. Its data shows an increase in the frequency
12. A vegetable vendor sells tomato 7 days a week and magnitude of extreme weather events since the
starting with Monday at the rate of Rs. 10, 5, 1950s. A separate analysis of European heatwaves
20, 25, 40, 25 and 50 per kg respectively. A revealed an increasing severity of such events over the
family spends a certain amount of money on past two decades. In the summer of 2022, southern
tomato per day from Monday to Wednesday, Europe experienced higher temperatures than usual for
and then spends half that amount on tomato on that time of the year. Spain, France and Italy saw daily
each of the next four days. The average maximum temperatures exceed 40°C. The EU’s
expense of the family on tomato (in rupee/kg) Copernicus Climate Change Service attributed these
over the week is closest to: unusually hot conditions to climate change and
(a) Rs. 12.12 suggested that such events are likely to become more
frequent, intense and last longer in the future —
(b) Rs. 10.40
indicating a concerning trend that may continue this
(c) Rs. 11
year.
(d) Rs. 13.60
www.visionias.in 4 ©Vision IAS
https://upscmaterial.online/

.
14. Which of the following statements can be 15. With reference to the above passage, consider
inferred from the above passage? the following statements.
1. Europe is witnessing the worst heatwave 1. In India, flooding is a frequent occurrence.
ever recorded. 2. The prompt relocation of individuals is
2. According to research, extreme weather made possible by non-structural flood
events are becoming more frequent and control techniques.
intense.
Which of the statements given above is/are
Select the correct answer using the codes
correct?
given below.
(a) 1 only
(a) 1 only
(b) 2 only (b) 2 only
(c) Both 1 and 2 (c) Both 1 and 2
(d) Neither 1 nor 2 (d) Neither 1 nor 2

Passage – 2 Passage – 3
In recent times, India has faced at least one major Today, the top three ingredients listed in the tiny print
flood event each year. As the 2023 monsoon on the backs of cans and bottles of Diet Pepsi — and
progresses, floods are beginning to repeat the pattern on its competitor Diet Coke — are water, caramel
of damage and destruction. Every year 1,600 lives on color and aspartame. A trip through the grocery store
average are lost due to floods, according to the reveals the ingredient on the labels of not only diet
National Disaster Management Authority. Floods also sodas but diet teas, sugar-free gums, sugar-free energy
affect 75 lakh hectares of land and inflict damage
drinks and diet lemonade drink mix. By some
worth Rs 1,805 crore to crops, houses and public
estimates, thousands of products contain aspartame.
utilities. A range of tools are available to manage
The use of aspartame, often known by the brand name
floods. Broadly, these are classified as structural and
Equal, in food and beverage products has long been
non-structural. Structural measures include storage
reservoirs, embankments, and diversions. These reduce scrutinised. The latest iteration came Thursday (July
flood hazards by keeping damage-causing waters away 13), when an agency of the World Health Organization
from agricultural areas, cities, industries, etc. Storage declared that aspartame could possibly cause cancer
reservoirs moderate the flood peak by storing water and encouraged people who consume a significant
during high-flow periods and releasing it after they number of beverages with aspartame to switch to water
have subsided. They also conserve water for irrigation, or other unsweetened drinks. But even with the
electricity generation, water supply, etc. Their emergence of many new artificial sweeteners, as well
effectiveness in flood moderation depends on the space as those that are plant- and fruit-based, Big Food just
available. In addition, tanks and ponds are traditional can’t quit aspartame, and analysts don’t expect it to
means of water conservation in India. They also aid in this time. That’s because the ingredient is one of the
groundwater recharge and promote biodiversity. Non- least-expensive sugar alternatives to use, it works
structural methods such as flood forecasting, warnings,
especially well in beverages and mixes, and people
and flood plain zoning, help in the timely evacuation
like the way it tastes. There was also pushback about
of people and regulate the use of floodplains. Note that
the urgency of the WHO’s announcement. In a quick
floods are a hazard only when people go close to flood
waters or hinder their movement. A forecasting and rebuke, the US Food and Drug Administration said it
warning system provides a priori estimate of disagreed with the findings, reiterating its stance that
approaching floods so that people and movable assets aspartame is safe. And a second WHO committee said
are relocated to safer places in time. India has more a 150-pound person would need to drink more than a
than 5,500 large dams. Accurate inflow forecasts help dozen cans of Diet Coke a day to exceed the safe
moderate floods by estimating the space needed for threshold for the sweetener.
storage in reservoirs.
www.visionias.in 5 ©Vision IAS

https://t.me/visioniastestseries2024
https://upscmaterial.online/

.
16. Which of the following statements are the 17. With reference to the above passage, consider
reasons that beverage firms may continue to the following statements.
use aspartame despite warnings? 1. CPTPP was the first pact that focused on
1. It is a cheaper sugar alternative . cutting trade barriers.
2. For aspartame to be carcinogenic,
2. CPTPP is the only pact having rules
consumption must be high, which is not
addressing competition, intellectual
typical for an ordinary person.
property rights and protections for foreign
3. Aspartame manufacturing company
promised to alter its chemical companies.
composition. Which of the statements given above is/are
Select the correct answer using the codes correct?
given below. (a) 1 only
(a) 1 and 2 only (b) 2 only
(b) 1 only (c) Both 1 and 2
(c) 2 and 3 only (d) Neither 1 nor 2
(d) 3 only

18. Which of the following can be attributed to


Passage – 4
“This is a modern and ambitious agreement and our CPTPP?
membership in this exciting, brilliant and forward- 1. Regulating Intellectual Property rights.
looking bloc is proof that the UK’s doors are open for 2. Safeguard against Chinese hegemony
business,” Badenoch said. She later told Sky News that across the world.
the pact meant that Britain has “a seat at the table in 3. Largest trading block after UK’s
the fastest-growing region” and that other countries are admission.
queuing up to join the pact. The CPTPP is a landmark Select the correct answer using the codes
pact agreed upon in 2018 that cuts trade barriers
given below.
among 11 countries. The pact requires countries to
(a) 1 only
eliminate or significantly reduce tariffs and make
(b) 2 and 3 only
strong commitments to opening services and
investment markets. It also has rules addressing (c) 3 only
competition, intellectual property rights and (d) 2 only
protections for foreign companies. CPTPP is seen as a
bulwark against China’s dominance in the region, 19. In an objective type question paper there are 5
although Beijing has applied to join, along with questions. The first two questions have three
Taiwan, Ukraine, Costa Rica, Uruguay and Ecuador. possible answers; the next two questions have
Politicians in several countries, including the UK and four possible answers and the last question has
Australia, are lobbying to keep China out, while
five possible answers. What is the total
Beijing is trying to prevent Taiwan from joining. The
number of ways in which all the five questions
UK government says CPTPP will cut tariffs for UK
exports to Asia Pacific countries and with UK could be answered?
membership, the trading bloc will have a combined (a) 1000
GDP of 12 trillion pounds and account for 15% of (b) 1280
global trade. Britain is keen to deepen trade ties in the (c) 720
Pacific after Brexit in 2020. (d) 405
www.visionias.in 6 ©Vision IAS

https://t.me/visioniastestseries2024
https://upscmaterial.online/

.
20. If n2 = 12345678987654321, what is 'n' ? 23. A Walmart outlet bought two kinds of cashew
(a) 12344321 - namely A and B. The price of A type was
(b) 1235789 thrice that of B type. It then sold 8 kg of B
(c) 111111111 type at a profit of 10% and 16 kg of A type at
(d) 1111111 a profit of 20% to a Spencer outlet. Thereafter,
Spencer lost 5 kg of A type and 3 kg of B type
21. The number of factors of a number ‘a’ is 12. in transit. It then mixed the remaining cashew
Which of the following cannot be the number together and sold the mixture at Rs. 166 per
of factors of a2? kg, making an overall profit of 25%. Which of
(a) 30 the following statements is true with respect to
(b) 33 the cost prices for Walmart?
(c) 23
(a) Price of A type cashew is more than Rs.
(d) 45
100.
(b) Price of B type cashew is less than Rs. 30.
22. Two statements S1 and S2 are given below
(c) Price of B type cashew is 4 less than the
followed by a question.
square of a whole number.
S1: a and b are prime numbers.
(d) Price of A type cashew is square of a
S2: b >7
whole number.

Question: Is a(b+2) an odd number, where a


24. A newspaper vendor finds that in a society
and b are distinct integers?
21% people read only Dainik Jagran, 18%
people read only The Telegraph, and 12% read
Which one of the following is correct in
only The Hindu. Moreover, some people read
respect of the above statements and the
exactly two newspapers - 8% read both Dainik
question?
Jagran and The Telegraph, 4% read both
(a) S1 alone is sufficient to answer the
Dainik Jagran and The Hindu, and 13% read
Question
both The Telegraph and The Hindu. Further,
(b) S2 alone is sufficient to answer the
340 people do not read any newspaper, while
Question
(c) S1 and S2 together are sufficient to 7% people read all the three newspapers. How

answer the question, but neither S1 alone many people read Dainik Jagaran?

nor S2 alone is sufficient to answer the (a) 1050

Question. (b) 1000

(d) S1 and S2 together are not sufficient to (c) 800

answer the Question (d) 900

www.visionias.in 7 ©Vision IAS

https://t.me/visioniastestseries2024
https://upscmaterial.online/

.
25. Two trains are travelling in opposite directions Directions for the following 3 (three) items:
Read the following two passages and answer the items
at uniform speeds of 90 km/h and 108 km/h that follow each passage. Your answers to these items
should be based on the passages only.
respectively. They take 10 seconds to cross
Passage – 1
each other. If the two trains had travelled in Chandrayaan-3, the mission that aims to achieve the
feat of conducting a “soft” landing on the Moon’s
the same direction, then a passenger sitting in southern pole, launched off from the Satish Dhawan
Space Centre (SDSC) in Andhra Pradesh on Friday
the faster moving train would have overtaken (July 14) afternoon. It will now take approximately 42
days to reach the Moon. Should it conduct a successful
the other train in 25 seconds. What is the landing, India will become only the fourth country –
after the United States, Russia, and China – to have
length of the faster train?
done so. While a successful launch is only the first step
(a) 400 m towards a long journey for the spacecraft, ISRO’s role
in carrying it through has seen great appreciation, and
(b) 450 m it has become a symbol of national pride for many.
That has been the case for long, such as in 2014, when
(c) 350 m the Mars Orbiter Mission (MOM) was inserted into the
Martian orbit. The low cost of the mission was also
(d) 425 m highlighted as an achievement. Since its inception in
1969, the country’s space agency ISRO has carried out
altogether 89 launch missions carrying satellites into
26. Which number should appear next in the series space. The history of space activities in India reflects
how little by little, institutions centered around space
8, 11, 9, 12, 10, 13, ..... exploration and research were set up and expanded.
Indian scientist EV Chitnis recounted in the book
(a) 8 From Fishing Hamlet to Red Planet: India’s Space
Journey, a compilation of essays from those involved
(b) 11
in this journey, that the first such organisation was the
(c) 12 Physical Research Laboratory set up in Ahmedabad,
Gujarat. Along with Vikram Sarabhai, a few scientists
(d) 13 worked here but lacked adequate funds. Chitnis recalls
putting together two boxes and an asbestos sheet as his
work table.

27. Pointing to Asha, John said, “The son of her 28. Consider the following statements.
1. A successful launch is the most important
only brother is the brother of my wife.” How step towards a long journey for the
spacecraft
is Asha related to John?
2. Only four countries have attempted to
(a) Grandmother reach the moon so far.
Which of the statements given above is/are
(b) Mother’s sister correct?
(a) 1 only
(c) Sister of father-in-law (b) 2 only
(c) Both 1 and 2
(d) Mother-in-law (d) Neither 1 nor 2
www.visionias.in 8 ©Vision IAS

https://t.me/visioniastestseries2024
https://upscmaterial.online/

.
Passage – 2 30. Why does history remember Marie
The French Revolution (1789-1799) not just changed Antoinette?
France forever, it had ripple effects across Europe and (a) For making fun of her poor subjects.
its colonies, shaping the concepts of democracy and (b) For being too generous as a queen.
equality as we know them today. One of the key (c) She knew very little about the real world
figures of the Revolution was Marie Antoinette, the
outside of palaces.
much-reviled Queen of France, who is still evoked as
(d) She wanted her subjects to lead a good
the epitome of insensitive profligacy. The statement
life.
that best sums up the Queen’s callously oblivious
personality is “Let them eat cake”. “…At a time of
31. Consider the following information, statement,
back-breaking inflation and record price rise, such
speeches are vulgar. In the late 18th century, France conclusions and answer the question that
was in deep economic crisis. When Marie Antoinette’s follows.
husband, Louis XVI, inherited the throne in 1774, the I. A + B means A is neither smaller than nor
treasury was already depleted, in a large part due to the equal to B.
decadence and corruption of the Paris elite. Louis XVI II. A – B means A is not greater than B.
providing help to the Americans in their war of III A × B means A is not smaller than B.
independence against the British did not help matters. IV. A ÷ B means A is neither greater than nor
In the 1780s, crops failed and prices rose to such an equal to B.
extent that even bread became unaffordable for many. V. A ± B means A is neither smaller than nor
It is in this context that the Queen, when told her
greater than B.
subjects could not afford the staple food of bread, is
believed to have said, then “let them eat cake”. The
Statement: P + Q, P × T, T + R, R ± S
massive folly of the sentence is that the Queen
apparently did not know that cake was more expensive
than bread, and that people who couldn’t afford the Conclusion-1: Q-T

latter could most definitely not buy the former. Conclusion-2: S±Q

29. Which of the following statements can be Which one of the following is correct in
inferred from the above passage? respect of the above Statement and the
1. The French Revolution helped shape the Conclusions?
modern democracy. (a) Only Conclusion-1 follows from the
2. Corruption of elite society members could Statement.
have severe economic consequences. (b) Only Conclusion-2 follows from the
Select the correct answer using the codes
Statement.
given below.
(c) Both Conclusion-1 and Conclusion-2
(a) 1 only
follow from the Statement.
(b) 2 only
(d) Neither Conclusion-1 nor Conclusion-2
(c) Both 1 and 2
follows from the Statement.
(d) Nether 1 nor 2
www.visionias.in 9 ©Vision IAS

https://t.me/visioniastestseries2024
https://upscmaterial.online/

.
32. If today is Friday, then which day would it be Directions for the following 5 (Five) items:
exactly on 1007th day from now? Read the following four passages and answer the
(a) Wednesday items that follow each passage. Your answers to these
(b) Tuesday items should be based on the passages only.
(c) Thursday Passage – 1
(d) Sunday
Foreign portfolio investors (FPIs) are driving up the
stock markets to new peaks on a daily basis, sending
33. The monthly income of Parul and Palak are in
investors into a buying euphoria. After renewed
the ratio 4:3 and the ratio of their expenditure
interest from FPIs helped the benchmark Sensex surge
is 3:2. If each of them saves Rs. 7000 then by
by around 10 per cent in the first quarter of fiscal
what percentage is Parul’s income more than
2024, the superfast Sensex has spurted 2.38 per cent,
that of Palak’s income?
in July so far even as analysts cautioned investors
(a) 25%
(b) 33.33% against entering into an overheated market. Domestic

(c) 50% stock markets continued their rally on Monday with

(d) Cannot be determined BSE Sensex, which closed above the 66,000-level last
week, hit a new high of 66,310.96 and the NSE Nifty
34. If 'PENCIL' is written as 'QGQGNR', then jumped to 19,641.90, a rise of nearly 0.30 per cent, in
how would 'ERASER' be written in that code early morning trades. The major driver is the return of
language? FPIs, the buoyancy in the global markets, strong
(a) FTDJWX macroeconomic fundamentals and the easing of
(b) FDTWJX inflation in India. “The scenario has changed with US
(c) FWTDJX
consumer inflation declining more-than-expected to
(d) FTDWJX
3% giving hopes that the US Fed is near the end of the
rate hiking cycle. Consequently, the US 10-year bond
35. Consider the sequence: To - - Ap - oP - A - -
yield has sharply dipped from 5.1% to 4.7% and the
oP - A- T- - lApT – Pl - p, which follows a
Dollar Index has crashed by nearly 4% from 103.57 to
certain pattern.
99.9. This is positive for emerging markets like India,

Which of the following set of letters completes which are likely to witness more capital flows,” said V

the given sequence? K Vijayakumar, Chief Investment Strategist at Geojit

(a) PltlpolpTpoa Financial Services. This resilience of the US

(b) PltpoplpTloa economy, which was not anticipated and discounted by


(c) PltlpTpoalpo the market, is the strongest pillar of support for the
(d) PlTlpTlpoPoA global markets now, he said.

www.visionias.in 10 ©Vision IAS

https://t.me/visioniastestseries2024
https://upscmaterial.online/

.
36. What among the following are responsible for 38. Which of the following statements best
the rise in the Indian stock market?
reflects what the passage implies?
1. Foreign Portfolio Investors (FPIs)
2. Decreasing trend of Inflation in India (a) Fight against corruption impacts mostly
3. Ceasing of the rate hike by the US Fed the top level; the ordinary man often
bank
continues to suffer.
4. Dollar index falling
Select the correct answer using the codes (b) Politicization of institutions and the lax
given below. attitude of citizens is the core reason for
(a) 1, 2 and 3 only
deep-rooted corruption.
(b) 2, 3 and 4 only
(c) 1, 3 and 4 only (c) Government must bring immediate
(d) 1, 2 and 4 only reforms in anti-corruption institutions.

(d) Without aware and informed citizens,


37. Which of the statements given below can be
inferred from the above passage? overcoming corruption is a difficult task.
1. The markets neglected the US economy's
resilient behaviour.
2. The surge in FPI has made India the Passage – 3
fastest-growing major economy. More than half of our country’s population resides in
Which of the statements given above is/are
villages. The women living here do not go for medical
correct?
(a) 1 only care during health problems or pregnancy because it is
(b) 2 only a temporary condition which will soon get better on its
(c) Both 1 and 2
own. This carelessness, which is prevalent even today,
(d) Neither 1 nor 2
is a major cause of India’s high maternal and infant
Passage – 2 mortality rates. Another point worth mentioning is the
While corruption remains endemic and deep-rooted,
female foeticide. The conventional thinking is that
India’s anti-corruption measures remain half-hearted
and slow. This is largely because the vital institutions since women have always been a burden, no good can
raised to fight graft lack genuine autonomy and a come out of a girl being born. These thinkers are
serious sense of purpose. Even a handful of anti-
corruption institutions (CVC, Lokpal) that enjoy some entirely incapable of grasping the horrors of a world
degree of autonomy, have not shown any signs of without women. If girls are born, they do not receive
being independent. However, fighting entrenched
the same commitment and care as boys would. Even
corruption should not be left to these handfuls of
macro or elite institutions alone. This is because, while though our constitution firmly guarantees free primary
corruption that happens at the top level often attracts schooling to everyone up to 14 years of age, only
media attention and the occasional national outrage, a
about 39 per cent of females in India can attend
great deal of corruption, which affects the ordinary
person, is at the retail level. primary schools.

www.visionias.in 11 ©Vision IAS

https://t.me/visioniastestseries2024
https://upscmaterial.online/

.
39. Which of the following statements best 40. Based on the above passage, the following
reflects the crux of the passage? assumptions have been made:
(a) The whole country should pledge to
1. Influx of multinational fast-food
improve the conditions of women in rural
companies has compromised the standard
areas by increasing awareness.
Indian dietary ecosystem.
(b) Even without attacking the patriarchal
mindset, ensuring women's empowerment 2. Indians may not stop eating Indian food

is possible. despite living in a foreign country.


(c) The road to women's empowerment in our Which of the above assumptions is/are valid?
country is blocked by the patriarchal
(a) 1 only
mindset and existing gender inequality in
(b) 2 only
rural society.
(c) Both 1 and 2
(d) Welfare of women is not their
responsibility alone, but also of society (d) Neither 1 nor 2

and the government.

41. Suppose a = (2/3) b, b = (2/3) c, and c = (2/3)


Passage – 4
d. What would be the value of b as a fraction
Although it has never had a standardized diet, India
of d?
has traditionally “imagined” its cuisine concerning the
incorporation and domestication of “foreign” (a) (2/3) d

influences. In the past two decades, with India (b) (4/3) d

becoming an economic powerhouse, a variety of (c) (4/9) d


multinational fast-food companies have entered the
(d) (8/27) d
previously protected Indian culinary landscape. They
include Pizza Hut, Mc- Donald’s, KFC, PepsiCo, and,
42. Town D is to the West of town M. Town R is
most recently, Taco Bell. These companies have had to
“Indianize” and self-domesticate to conquer the to the South of town D. Town K is to the East

notoriously difficult-to-please Indian palate. The of town R. Town K is in which direction from
Indian food market of $182 billion is believed to be town D?
growing at a rapid clip of 13 per cent. Indian
(a) South
precooked packaged foods empires such as MTR,
(b) East
SWAD, Haldirams, and Pataks have gone global,
(c) North – East
available wherever Indians now live, leading a quiet
yet unrecognized revolution in eating habits. (d) South – East
www.visionias.in 12 ©Vision IAS

https://t.me/visioniastestseries2024
https://upscmaterial.online/

.
Direction for the following 2 (two) Questions: 44. The vote value of the elected members of the
Study the graphs given below carefully and answer the Parliament (elected MPs) is calculated by the
formula:
2 (two) items that follow.
Value of vote of an elected MP =
In a Presidential election, following were the vote
value of each elected member (MLA) of some Indian
state legislative assemblies: It is given that the vote value of each elected
MP = 708 and the total value of votes of all
MLAs of all states is 549408.
If the elected members of the Parliament
include the elected members of the Lok Sabha
and the elected members of the Rajya Sabha
and if the number of elected members in the
Lok Sabha is 543, then what is the number of
elected members in the Rajya Sabha?
(a) 243
(b) 238
And following were the number of elected members in (c) 233
the respective state legislative assemblies: (d) 228

45. Pankaj and Suman are siblings with five


friends each. They are planning for a trip to
Kedarnath. Pankaj has 2 boy friends and 3
girl friends . Suman has 3 boy friends and 2
girl friends. In how many maximum possible
number of different ways can they invite 2
boys and 2 girls for the trip, such that two of
them are Pankaj’s friends and two are
Suman’s friends?
(a) 42
43. If the vote value of the elected members of the (b) 56
state legislative assembly (elected MLAs) is (c) 46
calculated by the formula: (d) 50
Value of vote of an elected MLA =
46. An English poetry book containing 282 pages
was torn by a baby. Now the book only has
pages numbered from 89 to 143. What must be
then what is the population of Uttar Pradesh? the total number of digits used in the entire
(a) 20.08 Crore book after it was torn?
(b) 12.08 Crore (a) 166
(b) 154
(c) 10.38 Crore
(c) 132
(d) 8.38 Crore
(d) 156

www.visionias.in 13 ©Vision IAS

https://t.me/visioniastestseries2024
https://upscmaterial.online/

.
47. Examine the following statements: 49. Ram and Shyam appeared in an examination.
1. I go to Goa only if I need leisure. There were a total of 10 questions in the exam,
2. I never need leisure when my family
carrying 1 mark each. There is a provision of
members are with me.
1/4th negative marking for every wrong
3. Whenever I go to Mumbai, I take my
answer. Ram attempted six questions while
family members along.
Shyam attempted seven. Ram scored 2.25 and

Which one of the following conclusions is Shyam scored 5.75 marks. What is the number

valid in the context of the above statements? of questions answered incorrectly by Ram and
(a) If I need leisure, I go to Goa. Shyam, respectively?
(b) If I need leisure, I seek my family (a) 3, 1
members’ company.
(b) 1, 3
(c) If I do not need leisure, I do not go to Goa.
(c) 2, 3
(d) If I am not with my family members, then
(d) Cannot be determined
I go to Goa.

48. Consider two solid substances A and B, which Directions for the following 6 (Six) items:
are made up of two ingredients X and Y. Read the following four passages and answer the
These two substances may or may not be
items that follow each passage. Your answers to these
contaminated by an impurity Z:
items should be based on the passages only.
Substance A:
Passage – 1
Ingredients Composition/Quantity
We've heard that health can improve a worker's
X 1/4 (one- fourth)
Y 3/4 (three- fourth) productivity by improving their physical capacity. But

Substance B: to be more accurate, improved health can improve a

Ingredients Composition/Quantity worker's ability to be more productive. Productivity is


X 1/6 (one- sixth) largely a factor of an employee's desire to work. Just
Y 5/6 (five- sixth) because they have the physical capacity to do more
If 2 kg of substance A and 3 kg of substance B
work doesn't necessarily mean they will. The human
were mixed to form another substance C, then
factor is simultaneously an organization's greatest asset
the ratio of X and Y in the substance C is:
and its greatest liability. High-performing, healthy and
(a) 1:6
(b) 1:4 productive employees are the gears that drive a

(c) 5:12 business whilst a dive in employee productivity applies


(d) 1:12 brakes to those gears ultimately slowing profits.

www.visionias.in 14 ©Vision IAS

https://t.me/visioniastestseries2024
https://upscmaterial.online/

.
50. Which one of the following statements best Passage – 3
reflects the crux of the passage? Though investment in EdTech has been increasing,
(a) For enhanced profits, organizations can
learning and outcomes as a result have not changed
promote programs for improving the
health of their employees. considerably in many countries. An OECD report
(b) For an organization, a motivated employee found that when it comes to the impact of computer
is as important as a healthy employee.
usage in schools as measured through PISA, “impact
(c) Productivity of an employee is directly
related to his health. on student performance is mixed, at best." COVID

(d) Organizations should explore mechanisms experience to date highlights that teaching and learning
to enhance the employee’s desire to work. remotely is not the same as face-to-face
pedagogy. While we can never replace the magic that
Passage – 2
Climate change is a global phenomenon that largely happens between great teachers and students in an in-
impacts urban life. Rising global temperatures cause person environment, we should focus on the social
sea levels to rise, increase the number of extreme
aspects of technology to enhance connections from a
weather events such as floods, droughts, and storms,
distance. Much more attention must be directed to how
and increase the spread of tropical diseases. All these
have costly impacts on cities' basic services, technology will enhance teaching and learning in a
infrastructure, housing, human livelihoods, and health. blended learning environment reaching students, both
At the same time, cities are a key contributor to
in school and at home.
climate change, as urban activities are major sources of
greenhouse gas emissions. Estimates suggest that cities
are responsible for 75 per cent of global CO2 52. Which of the following is/are the most rational
emissions, with transport and buildings being among and logical inference/inferences that can be
the largest contributors.
made from the passage?

51. Which one of the following statements best 1. Usage of technology leads to improved
reflects the crux of the passage? educational outcomes for students.
(a) People in urban areas face the double
2. Teachers with good emotional intelligence
whammy of climate change and unplanned
urbanization. can enhance connections with students
(b) Development of rural areas will reduce the from a distance.
emissions caused by transportation sector Select the correct answer from the code given
and buildings.
below.
(c) Controlled urbanization will help reduce
the impact of climate change on the people (a) 1 only
of urban areas. (b) 2 only
(d) Urbanization exacerbates the impacts of
(c) Both 1 and 2
climate change on the people living in
(d) Neither 1 nor 2
urban areas.

www.visionias.in 15 ©Vision IAS

https://t.me/visioniastestseries2024
https://upscmaterial.online/

.
53. With reference to the passage, which one of 54. Based on the above passage, the following
the following statements is correct? assumptions have been made:
(a) Technology ensures inclusive growth of
1. Lack of understanding about privacy
the students in the education sector.
among the youth could be a major reason
(b) Technology should empower students not
behind cybercrimes.
only in remote, but in in-person
environments as well. 2. Misuse of information may happen

(c) With the potential of technology in partially because of the slowly evolving
changing experiences in the education legal provisions.
sector, it should be deployed to replace the Which of the above assumptions is/are
in-person environment to a certain extent.
invalid?
(d) Teaching quality cannot improve by
(a) 1 only
giving teachers access to technology.
(b) 2 only

Passage – 4 (c) Both 1 and 2

Social media is a form of communication via the (d) Neither 1 nor 2


Internet. Its main goal, when it came into being, was to
create a virtual kinship network throughout the world. 55. What is the most rational inference that can be
The users of social networking sites were untroubled
drawn from the passage?
until the coming of the 1990s. This was when
(a) The most important step to controlling
cybercrime was born. Believe it or not, it is we who
give away our personal information online. cybercrimes is to upgrade the cyber

Intentionally or unintentionally, we give away a lot of security software, protocols and


our personal information. Tons of cyber information algorithms.
available online has opened the gates for new legal (b) Social media is the prime medium
challenges for which adequate laws are yet to be
contributing to the misuse of personal
framed. These can range from stealing your social
information of the youth.
security benefits, filing compensation claims using
(c) Mere cybersecurity solutions will not stop
your credentials and using your names for making
monetary transactions in their name to using your the misuse of the personal information

credentials for making fake passports, PAN cards etc. online.


It has been noted that the younger generation falls prey (d) The overall improvements in cybersecurity
to such cybercrimes the most. This is majorly because should not be limited to present-day
of their immaturity, which is easily identified by these
requirements, but address future needs as
criminal minds. Though we wrongly end up accusing
well.
the cybersecurity loopholes in many cases.
www.visionias.in 16 ©Vision IAS

https://t.me/visioniastestseries2024
https://upscmaterial.online/

.
56. Neil Armstrong was the first person to walk on 58. In an army platoon, each soldier carries a
the Moon. He had to install 6 substations, and different number of hand grenades with him,
all the apparatus required to install a substation
starting from one. Also, no one carries more
were stored in one main station. The location
of 1st substation was 5 jumps away from the hand grenades than the number of soldiers in
main station, the location of 2nd substation was the platoon. If no soldier carries 55 hand
6 jumps away from 1st substation, the location
grenades, then find the maximum possible
of 3rd substation was 7 jumps away from 2nd
number of soldiers in that particular platoon.
substation and so on. To install a substation,
he carried the apparatus from the main station (a) 54
to the substation, installed the apparatus at the (b) 55
substation, and then returned to the main
(c) 27
station. It is known that one jump of a man on
earth is equal to 1 meter, while on moon one (d) 28
jump of man is equal to 5 meters. If he
installed the 6th substation first, followed by 5th 59. Tushar was travelling to Mumbai from Delhi.
substation and so on and then finally the 1st
When he left Delhi on Friday noon, his watch
substation, and returned to main station, then
how many meters did Neil Armstrong travel to was slow by 2 minutes. He took rest at
install all the 6 substations? Gwalior and got to know that his watch
(a) 1400 metres showed correct time at that time. He reached
(b) 1000 metres
Mumbai at 2 P.M. on the following Friday and
(c) 280 metres
(d) 200 metres saw that his watch is 4 minutes 48 seconds
fast. If the watch gains uniformly, then when
57. An important bill was introduced in the
did he reach Gwalior?
Parliament. A total of 500 parliamentarians
(a) 2 P.M. on Sunday
voted on it and the bill was passed. However,
after some allegations on the voting process, (b) 2 P.M. on Saturday
the bill was put to vote again. In the second (c) 12 P.M. on Monday
round of voting, the number of opponents
(d) 12 P.M. on Sunday
were increased by 100% as compared to the
initial round. This time the number of
supporters was one-third of the first round. 60. 343 identical cubes are arranged to form a
How many parliamentarians voted against the cubical block. How many of these cubes must
bill initially in the first round, if it’s known
be surrounded by other cubes from all sides?
that no one abstained from the voting in either
round? (a) 127
(a) 300 (b) 125
(b) 200
(c) 121
(c) 250
(d) 18
(d) 400
www.visionias.in 17 ©Vision IAS

https://t.me/visioniastestseries2024
https://upscmaterial.online/

.
61. There are 8 books of Bengali, Tamil, Hindi 62. Which of the following statements best
and Marathi arranged in a stack. The reflects what the passage implies?
(a) Like other professions, the inclusion of
information regarding their relative placement
women's perception in the judiciary should
is given below: be a priority.
1. Hindi book is below the Marathi books. (b) Justice delivery should not suffer from
2. Two books of Tamil are between the two lopsided gender notions.
(c) Due to the lack of representation of
books of Marathi.
women in the judiciary, justice for women
3. One book of Tamil is just between the two
is delayed.
books of Bengali. (d) For integrating the gender perspective and
4. On counting from top it is found that, the giving equal visibility to women, women
second, fifth and sixth books are of Tamil. judges should hear cases of women.

Passage – 2
Which book is fifth from below? We all remember the famous dialogue of the movie “3
(a) Bengali Idiots”. Life is a race. Suppose you don’t run fast. You
(b) Tamil will be like a broken egg. But there is a fundamental
question - what happens when you fall and die because
(c) Hindi
of running fast in that race? Every parent wants their
(d) Marathi child to be successful in life. Be a winner, be a topper.
That is what everybody wants the school, teachers,
Directions for the following 4 (four) items: parents, or even our relatives. They all bear well in
their hearts. But sometimes, they must remember to
Read the following three passages and answer the
evaluate whether children are motivated or stressed.
items that follow each passage. Your answers to these
Academic pressure causes students to feel more
items should be based on the passages only. stressed and anxious, negatively affecting their
Passage – 1 physical, social, and emotional well-being.
When judges interpret and implement the law, their
63. Which of the following statements best
reasons and opinions reflect their thought process, an
reflects the crux of the passage?
insight into their perceptions. These perceptions at the (a) School, teachers, parents, or even our
very least must be representative of both men and relatives bear well in their heart as they
women on the bench to ensure a fair and adequate pressurize the children unintentionally.
(b) Performing well despite severe academic
response through judicial decisions. It is important to
pressure reflects the mettle of a child.
note that including women in the judiciary is not (c) Intrinsic drive with fair expectations
simply about ensuring that her perception is relevant to would not stress a child in pursuit of
resolving cases about women. It is much more than excellence in academics.
(d) For the sake of the happiness of children,
that. It is about integrating the gender perspective and
parents should not pressurize students to
giving equal visibility to women.
perform well in academics.
www.visionias.in 18 ©Vision IAS

https://t.me/visioniastestseries2024
https://upscmaterial.online/

.
Passage – 3 66. Which of the following expressions in not
India holds the second position in the world for equivalent to p - 2/3(2p - 3q) - 1/3(p + 4q) ?
freshwater fish-farmed production. There is immense (a) (-2/3) (p - q)
scope for its development when improved systems and
(b) (-2/3)p + (2/3)q
species are adopted. A large extent of Indian
(c) (-2/3) (p + q)
aquaculture is still based on traditional farming
methods. Converting them to modern farming methods (d) ( -1/3) (2p - 2q)
will increase fish production and will also address the
sustainability of the environment. The marine fish 67. Consider the figure given below:
farming segment is hitherto untapped and establishing
commercial hatchery technologies and suitable
farming methods is yet another significant opportunity,
given the country’s vast coastline.

64. Based on the above passage, the following


assumptions have been made:
1. Adoption of commercial hatchery
technologies could make India the
number one in freshwater fish-farmed
What is the sum of x and y in the figure given
production.
2. Modern fish farming methods are more above?
environmentally friendly than the (a) 105
traditional farming approaches. (b) 101
Which of the above assumptions is/are valid? (c) 99
(a) 1 only
(d) 108
(b) 2 only
(c) Both 1 and 2
(d) Neither 1 nor 2 68. A shop offers a 15% discount on the marked
price on a regular basis. During festive
65. Which of the following statements best seasons, it offers another discount of 10% over
reflects the crux of the passage? and above the regular discount, and the owner
(a) The potential for aquaculture in India has
still earns a profit of 8.49%. What would have
not been realized to its fullest potential.
been the profit percentage if only a 7%
(b) Marine fish farming in India is more
beneficial than freshwater fish farming. discount is offered over and above the regular
(c) India’s marine environment acts as a discount?
limitation for it to become the leader in (a) Between 11% and 13%
fish production. (b) Between 9% and 11%
(d) Any change in technology concerning
(c) Between 13% and 15%
fishing must consider its environmental
(d) Between 15% and 18%
impact.

www.visionias.in 19 ©Vision IAS

https://t.me/visioniastestseries2024
https://upscmaterial.online/

.
Directions for the following 2 (two) items: Directions for the following 3 (three) items:
Read the following information and answer the
Read the information given below and answer the two
questions that follow.
items that follow. A, B, C, D, E, F and G are seven persons who travel to
office everyday by a particular train which stops at five
Five teams – A, B, C, D and E participated in a
stations 1, 2, 3, 4 and 5 respectively after leaving its
tournament. Each of the five teams played exactly one base station. Further, it’s known that:
1. Three among them get on the train at the base
match with other teams. No two teams won the same
station.
number of matches, and no match ended in a draw/tie. 2. D gets down at the next station from the one at
which F gets down.
It is also known that:
3. B does not get down either with A or F.
(i) The number of matches lost by A was an odd 4. G alone gets on at station 3 and gets down with C
after having passed one station.
number; the same was true for D.
5. A travels between only two stations (i.e. only from
(ii) E ended up with minimum number of wins. one station to another with no station falling in
between) and gets down at station 5. He is the only
(iii) The number of matches won by D was more than
one boarding the train at his station.
that by B. 6. None of them gets on at station 2.
7. C gets on with F but does not get on with either B
or D.
69. Find the number of matches lost by B, C and D 8. E gets on with two others and gets down alone at a
station that is subsequent from the one at which D
put together.
gets down.
(a) 2 9. B and D work in the same office and they get
down together at station 3.
(b) 3
10. None of them gets down at station 1.
(c) 4

(d) 5 71. At which station do both C and F get on?


(a) 1
(b) 2

70. Which team won the maximum number of (c) 4


(d) Cannot be determined
matches?

(a) A 72. After how many stations does E get down after
boarding the train?
(b) B (a) One
(c) C (b) Two
(c) Four
(d) D (d) Five
www.visionias.in 20 ©Vision IAS

https://t.me/visioniastestseries2024
https://upscmaterial.online/

.
73. At which station did the maximum people get 76. Which one of the following statements best
down? reflects the crux of the passage?
(a) 2
(a) The value of stolen artefacts cannot be
(b) 3
(c) 4 measured because of their historical
(d) 5 importance.
(b) Inspite of the global nature of stolen
74. If 3 * 4 $ 5 = 54, 2 * 3 $ 5 = 48, then 5 * 6 $ 7 artefacts issue, India has seen some
=?
success in its efforts to recover them.
(a) 130
(b) 65 (c) There is a need for a global consensus and
(c) 90 information sharing about stolen artefacts
(d) 195 for their return to their home country.
(d) Mere law formulation is not enough to
75. Anuj started a business by investing a certain
amount. After few months, Rakesh joined him control the smuggling of stolen artefacts;
by investing 33.33% more amount than Anuj. effective enforcement is a must.
After 12 months, the share of Anuj is Rs. 9216
out of a total profit of Rs. 17408. What is the
Passage – 2
ratio of time periods of Anuj and Rakesh?
Throughout history, people and societies have adjusted
(a) 2 : 3
(b) 3 : 2 to and coped with changes in climate and extremes
(c) 1 : 2 with varying degrees of success. Climate change
(d) 3 : 4 (drought in particular) has been at least partly
responsible for the rise and fall of civilizations. Earth’s
Directions for the following 3 (three) items:
Read the following two passages and answer the items climate has been relatively stable for the past 10,000
that follow each passage. Your answers to these items years, and this stability has allowed for the
should be based on the passages only. development of our modern civilization and
Passage – 1
agriculture. Our modern life is tailored to that stable
The theft of historically valuable objects did not stop
with the end of colonialism. The smuggling of climate, not the much warmer climate of the next
artefacts remains a pressing concern to date. The thousand years. As our climate changes, we will need
illegal trade in smuggled antiques is a profitable to adapt. The faster the climate changes, the more
venture for smugglers. It has become a global concern difficult it will be. While climate change is a global
today. Many artefacts from India have been stolen in
issue, it is felt on a local scale. Local governments are
the decades after Independence. Often, these surface
later in various countries. Such items obtained by therefore at the frontline of adaptation. Cities and local
fraudulent means are sold for exorbitant amounts of communities around the world have been focusing on
money. To combat this problem, India passed the solving their climate problems. They are working to
Antiquities and Art Treasures Act in 1972 for the
build flood defences, plan for heat waves and higher
protection and preservation of valuable objects of the
past. The country's struggles for conserving and temperatures, install better-draining pavements to deal
recovering its artefacts have seen considerable success with floods and stormwater, and improve water storage
so far. Nevertheless, there is still a long journey ahead. and use.

www.visionias.in 21 ©Vision IAS

https://t.me/visioniastestseries2024
https://upscmaterial.online/

.
77. Based on the above passage, the following 79. Consider the following Statements and
assumptions have been made: Conclusions:
1. In the process of climate change Statements:
adaptation, the role of local governments S1: Only Graduates are Clerks.
is more important than national S2: Some Graduates are Officers.
governments. S3: Some Officers are Scientists.
2. Common people have had a minimal role
in accelerating climate change across the Conclusions:
earth. I. No Clerk is an Officer.
Which of the above assumptions is/are valid? II. All Clerks being Scientists is a possibility.
(a) 1 only III. All Officers are Graduates.
(b) 2 only
(c) Both 1 and 2 Which of the above conclusion(s) can be
(d) Neither 1 nor 2 drawn from the given statements?
(a) All I, II and III
78. Which of the following statements best (b) Only I and II
reflects the crux of the passage? (c) Only II
(a) Considering the possible devastation of (d) Only II and III
climate change, adaptations at the local
level can help our modern lives to 80. A property dealer decided to sell his 100
continue with stability. newly build flats. The cost of each flat is Rs.
(b) Despite being a global issue, climate 25 lakhs. Flats are numbered from 1 to 100.
change requires interventions at the local Persons who were allotted flat number which
level for its reversal. is divisible by 8 but not by 6 got 5% discount.
(c) For a stable modern life, people should not Find the number of persons who got this 5%
only focus on local adaptation but also discount.
work towards global mitigation of climate (a) 4
change. (b) 8
(d) Unless the adaptations at the local level (c) 10
are done, it would be challenging to tackle (d) 12
climate change at the global level.

Copyright © by Vision IAS


All rights are reserved. No part of this document may be reproduced, stored in a retrieval system or transmitted
in any form or by any means, electronic, mechanical, photocopying, recording or otherwise, without prior
permission of Vision IAS.

www.visionias.in 22 ©Vision IAS

https://t.me/visioniastestseries2024
https://upscmaterial.online/

.
VISIONIAS
www.visionias.in
ANSWERS & EXPLANATION
CSAT 2024 – ABHYAAS TEST – 3 – 5184

1 (c)
According to the question, among the girls, Tanu is at 4th position from the beginning and 6th position from the
end.

So, Total number of girls = (6 + 4) - 1 = 9

Among the boys, Atul is at 7th position from the beginning and 24th position from the end.

So, Total number of boys = (24 + 7) – 1 = 30

So, Total number of children = 30 + 9 = 39


Hence, option (c) is the right answer.

2 (c)
From A and B,
SI = Prn/100
Or 4500 = (P × 10 × 3)/100
Or P = 15000
So, we can calculate CI using the formula: CI = P(1 + r/100)n - P

From B and C,
Difference between CI and SI after 3 years = PR2 (300 + R)/1003
Or P = 465000000/(100 × 310)
Or P = 15000
So, we can calculate CI using the formula: CI = P(1 + r/100)n - P
Hence, option (c) is the right answer.

1 www.visionias.in ©Vision IAS

https://t.me/visioniastestseries2024
https://upscmaterial.online/

.
3 (a)
Let M be the daily wage of a male and F be the daily wage of a female.
According to the question,
15M + 25F = 5800 ………(ii)
20M + 30F = 7200 ……….(ii)

By subtracting equation (i) from (ii), we get:


5M + 5F = 1400
Or M + F = 280
Or M = 280 – F ……….(iii)

From equations (ii) and (iii), we get:


20(280 – F) + 30F = 7200
Or 5600 – 20F + 30F = 7200
Or 10F = 7200 – 5600
Or 10F = 1600
Or F = 160
Therefore, the total wage received by 10 females in one day = 160 × 10 = Rs. 1600
Hence, option (a) is the right answer.

4 (c)
Let Jabalpur, Kolkata, Lucknow, Mumbai, Nagpur, Orangabad, Panipat and Qarimnagar be represented by J, K, L,
M, N, O, P and Q respectively.

One-way route is denoted by a single side arrow,


Two-way route is denoted by a double side arrow.

The diagram as per the given data is as follows:

As per the diagram, Statement 1 is correct, because she doesn’t need road route to reach Mumbai from Orangabad.

Statement 2 is also correct, because Sachin will visit three cities in the longest possible route, viz Jabalpur, Panipat
and Orangabad (or Qarimnagar, Panipat and Orangabad) before reaching Kolkata.
Hence, option (c) is the right answer.
2 www.visionias.in ©Vision IAS

https://t.me/visioniastestseries2024
https://upscmaterial.online/

.
5 (c)
According to the question, Eva (E) sends the problem to Bablu (B), i.e. E → B

Using Statement-1: Appu(A) is the last person to see the problem. And, Cibbu takes up the problem only after
Bablu. So, we get:
Bablu (B) … → … Cibbu (C) … → … Appu(A)

Using Statement-2: Bablu or Eva takes up the problem only after Dabbu
So, Dabbu (D) … → … B/E

On combining above pieces of information, we get:


D → E → B → C→ A
Hence, option (c) is the right answer.

6 (b)
Present age of younger child = x years
Present age of older child = (x + 8) years
Sum of the present ages of 4 member family = (4 × 40 + 4× 15) years = (160 + 60) years = 220 years
Sum of present ages of 6 members = 6 × 40 = 240 years
∴ Sum of the present ages of children = 240 – 220 = 20 years
∴ x + x + 8 = 20
Or 2x = 20 – 8 = 12
Or x = 6 years
∴ Present age of older child = 6 + 8 = 14 years
∴ Required ratio= 14 : 6 = 7 : 3
Hence, option (b) is the correct answer.

7 (b)
Numeral ‘v’ appears on pages iv, v, vi, vii and viii, i.e. 5 times from i to x and another 5 times from xi – xx. Thus,
it will appear 10 times.
However, numeral ‘x’ appears on pages ix, x, xi, xii……… .... xx, i.e. on 12 pages.
Hence, option (b) is the correct answer.

8 (d)
Let the cost of a pen, an eraser and a sharpener be represented by p, e and s respectively.
According to the question,
2p + e + 3s = 23 ………(1)
6p + 3e + s = 45 ……….(2)
14p + 7e + 21s = 161 …….(3)
Here, on observing equations 1 and 3, we can conclude that they are the same. Equation (3) is 7 times that of
equation (1).
3 www.visionias.in ©Vision IAS

https://t.me/visioniastestseries2024
https://upscmaterial.online/

.
So, we have three variables but only two equations. Hence, we cannot determine the value of all the three
variables. Though using equations (1) and (2) we can determine the value of s, but not that of p and e. Thus option
(d) is correct.

9 (d)
Number of candidates which failed in English only = 31 – 22 = 9%
Number of candidates which failed in Maths only = 35 – 22 = 13%
So, Number of candidates which failed in at least one subject = 9% + 13% + 22% = 44%
So, Statement 1 is incorrect.

Number of candidates which passed in both the subjects = 100 - 44 = 56%


Number of candidates which passed in English = 100 – 31 = 69%
Number of candidates which passed in English only = 69 – 56 = 13%
Number of candidates which passed in English only = 13% of 1,66,000 = 13 × 1,66,000/100 = 21,580
Thus, neither Statement 1 nor Statement 2 is correct.
Hence, option (d) is correct.

10 (d)
Since, the combined average marks of the students of L2 and L3 batches together is equal to the average marks of
the students in four batches viz. L1, L2, L3 and L4, therefore, the combined average marks of the remaining two
batches (L1 and L4) must also be equal i.e. 64%.

Now, we know that the individual average marks of the students of L1 and L4 batches are 60% and 80%
respectively. Applying Allegation rule, we get:

Thus, the ratio of the number of students in batches L4 and L1 is 1 : 4.


Hence, option (d) is correct.

4 www.visionias.in ©Vision IAS

https://t.me/visioniastestseries2024
https://upscmaterial.online/

.
11 (a)

There are a total of 24 triangles in the above figure, namely ABC, CDE, ENF, FGH, HIJ, JMA, CJA, CHJ, AHJ,
ACH, ACK, CHK, HJK, AKJ, CEH, CEF, EFH, CHF, CEL, EFL, FHL, HCL, CJF, and AEH.
Hence, option (a) is correct.

12 (a)
Let the daily expense on the first three days be Rs. 100.
So, for the next four days the spending on tomato would be half of the first three days’ daily spending, i.e. 100/2 =
Rs. 50.

Days Amount Price/kg Quantity bought (Amount/Price per kg)

Monday 100 10 100/10 = 10 kg

Tuesday 100 5 100/5 = 20 kg

Wednesday 100 20 100/20 = 5 kg

Thursday 50 25 50/25 = 2 kg

Friday 50 40 50/40 = 1.25 kg

Saturday 50 25 50/25 = 2 kg

Sunday 50 50 50/50 = 1 kg

Total 500 41.25 kg

So, overall average expense on tomato over the week = 500/41.25 = Rs. 12.12
Hence, option (a) is the correct answer.

13 (b)
Let the time taken by her to cover the journey (wherein she reaches the office on time) be x minutes.

When Dipti runs at 30 kmph, she takes (x + 17) minutes.


So, Distance = 30(x + 17)/60 km

When she runs at 40 kmph, she takes (x + 5) minutes.


So, Distance = 40(x + 5)/60 km
5 www.visionias.in ©Vision IAS

https://t.me/visioniastestseries2024
https://upscmaterial.online/

.
Distance is the same in both the cases.
So, 30(x + 17)/60 = 40(x + 5)/60
Or 30x + 510 = 40x + 200
Or 310 = 10x
Or x = 31 minutes
Hence, option (b) is the correct answer.

14 (b)
Statement 1 is not correct. The passage does not mention that this is the worst ever recorded heatwave. Although
it says, “Europe is currently in the midst of a heatwave. Italy, in particular, is expected to face blistering heat,
with temperatures projected to reach 40℃ to 45℃. There’s even a chance that the current European temperature
record of 48.8℃, set in Sicily in 2021, could be surpassed”, it cannot be inferred that it is the worst-ever heatwave
recorded in Europe. Hence, it is not a correct statement.
Statement 2 is correct. The passage says, “Research by the Intergovernmental Panel on Climate Change
confirms this trend. Its data shows an increase in the frequency and magnitude of extreme weather events since
the 1950s”. The underlined portion highlights the increase in frequency and intensity of heatwaves. The passage
further says, “A separate analysis of European heatwaves revealed an increasing severity of such events over the
past two decades. …. such events are likely to become more frequent, intense and last longer in the future … ”. It
can be inferred that extreme weather events are becoming more frequent and intense. Hence, it is a correct
statement.

15 (c)
Statement 1 is correct. India has recently had at least one significant flood each year. Floods are starting to
replicate the pattern of havoc and destruction as the 2023 monsoon progresses. The passage says, “Every year
1,600 lives on average are lost due to floods, according to the National Disaster Management Authority”. It can
be inferred that floods occur frequently every year. Hence, it is a correct statement.
Statement 2 is correct. The passage says, “Non-structural methods such as flood forecasting, warnings, and
flood plain zoning, help in the timely evacuation of people and regulate the use of floodplains. Note that floods are
a hazard only when people go close to flood waters or hinder their movement. A forecasting and warning system
provides a priori estimate of approaching floods so that people and movable assets are relocated to safer places
in time”. This implies that Non-structural methods aid in the prompt relocation of people and movable assets to
safer locations by providing a priori alarm of impending floods. Hence, it is a correct statement.

16 (a)
Statement 1 is correct. The passage clearly states, “But even with the emergence of many new artificial
sweeteners, as well as those that are plant- and fruit-based, Big Food just can’t quit aspartame, and analysts
don’t expect it to this time. That’s because the ingredient is one of the least-expensive sugar alternatives to use”.
Hence, it can be inferred that aspartame users will continue to use it as it is a cheaper alternative. Hence, it is a
correct statement.
Statement 2 is correct. The passage clearly says, “…a second WHO committee said a 150-pound person would
need to drink more than a dozen cans of Diet Coke a day to exceed the safe threshold for the sweetener”. The tone
of the passage indicates that usually people consume aspartame within safe limits. Hence, it is a correct statement.
Statement 3 is not correct. It is nowhere mentioned in the passage that aspartame manufacturing companies have
made a commitment to alter its chemical composition. Hence, it is not a correct statement.

6 www.visionias.in ©Vision IAS

https://t.me/visioniastestseries2024
https://upscmaterial.online/

.
17 (d)
Statement 1 is incorrect. Refer to the line: “The CPTPP is a landmark pact agreed upon in 2018 that cuts trade
barriers among 11 countries. The pact requires countries to eliminate or significantly reduce tariffs…” The
author only says that CPTPP aims at cutting trade barriers. There is no indication on whether it is the first such act
or not. Therefore, this answer option is incorrect.
Statement 2 is incorrect. Rules for addressing competition, intellectual property rights and protections for foreign
companies are some of the components of this pact. However, we cannot say whether or not it is the only pact
having these components. Therefore, this answer option is incorrect.

18 (a)
Statement 1 is correct. The CPTPP addresses multiple issues. The passage says, “It also has rules addressing
competition, intellectual property rights and protections for foreign companies.” Hence, it is a correct statement.
Statement 2 is not correct. The CPTPP will protect against Chinese hegemony in the Asia Pacific region and not
at the global level. The passage says, “CPTPP is seen as a bulwark against China’s dominance in the region”.
Hence, it is not a correct statement.
Statement 3 is not correct. The passage does not mention that CPTPP will become the largest trading block after
UK entry. The passage says, “…with UK membership, the trading bloc will have a combined GDP of 12 trillion
pounds and account for 15% of global trade”. The block will get a substantial boost after UK’s entry. Whether it
will become the largest trading block is not clear. Hence, it is not a correct statement.

19 (c)
Total number of possibilities for the first two questions = 3 × 3 = 9.
Total number of possibilities for the next two questions = 4 × 4 = 16.
Total number of possibilities for the last question = 5
Total number of ways in which all the five questions could be attempted = 9 × 16 × 5 = 720
Hence, option (c) is the correct answer.

20 (c)
Square root of 121 =11
Square root of 12321 = 111
Square root of 1234321 = 1111
Square root of 123454321 = 11111
Square root of 12345654321 = 111111
Square root of 1234567654321 = 1111111
Square root of 123456787654321 = 11111111
So, square root of 12345678987654321 = 111111111.
Hence, option (c) is correct.

21 (a)
It is given that ‘a’ has 12 factors. So, we can express ‘a’ in the following ways:
If a = x11, the number of factors of a is (11 + 1), i.e. 12.
a2 = x22, so the number of factors of a2 is (22 + 1), i.e. 23.

7 www.visionias.in ©Vision IAS

https://t.me/visioniastestseries2024
https://upscmaterial.online/

.
If a = x5y, the number of factors of a is (5 + 1) × (1 + 1), i.e. 12.
a2 = x10y2, so the number of factors of a2 is (10 + 1) × (2 + 1), i.e. 33.

If a = x3y2, the number of factors of a is (3 + 1) × (2 + 1), i.e. 12.


a2 = x6y4, so the number of factors of a2 is (6 + 1) × (4 + 1), i.e. 35.

If a = xyz2, the number of factors of a is (1 + 1) × (1 + 1) × (2 + 1), i.e. 12.


a2 = x2y2z4, so the number of factors of a2 is (2 + 1) × (2 + 1) × (4 + 1), i.e. 45.
Hence, 30 cannot be the number of factors of a2.
Hence option (a) is the correct answer.

22 (d)
From S1: a and b are prime numbers.
If any of a and b is 2, a(b+2) will be an even number.
On the other hand, if both are odd prime numbers, a(b+2) will also be an odd number.
So, S1 alone is not sufficient.

From S2: b >7


This is clearly not sufficient, as we don’t know anything about a.
So, S2 alone is not sufficient.

Using S1 and S2 together:


If a = 2, then a(b+2) will be even, and if a is an odd prime number, then a(b+2) will be odd.
Thus, the question cannot be answered even by using both the statements together.
Hence option (d) is the correct answer.

23 (c)
Let the cost prices for Walmart for A type cashew and B type cashew be Rs. 3x per kg and Rs. x per kg
respectively.
It sold 8 kg of B type at a profit of 10% and 16 kg of A type at a profit of 20% to a Spencer outlet.
So, Cost price of B type cashew for Spencer = Rs. 1.1x per kg
And, Cost price of A type cashew for Spencer = 1.2 × 3x = Rs. 3.6x per kg
So, Overall Cost Price for Spencer = Rs. (3.6x × 16 + 1.1x × 8)

Spencer lost 5 kg of A type and 3 kg of B type in transit.


So, Remaining quantity of A type cashew = 16 - 5 = 11 kg
Remaining quantity of B type cashew = 8 - 3 = 5 kg
Total cashew left = 11 + 5 = 16 kg

8 www.visionias.in ©Vision IAS

https://t.me/visioniastestseries2024
https://upscmaterial.online/

.
It mixed the remaining cashew together and sold the mixture at Rs. 166 per kg, making an overall profit of 25%.
So, Selling Price = (125/100) × Cost Price = (5/4) × Cost Price
or, (5/4) × Cost Price = Selling Price
or, (3.6x × 16 + 1.1x × 8) × (5/4) = 16 × 166
or, 83x = 16 × 166
or, x = (16 × 166)/83 = Rs. 32
Now, 32 is 4 less than the square of 6 (i.e. 36).
Hence, option (c) is the correct answer.

24 (c)
On representing the given data via a Venn diagram, we get:

Number of people who don’t read any newspaper= 100% - {21 + 12 + 18 + 4 + 13 + 8 + 7}%
= 100% - 83% = 17%

Now, we know that 340 people don’t read any newspaper.


Hence, total number of people in the society = (340/17) × 100 = 2,000

Percentage of people that read Dainik Jagran = 7 + 4 + 8 + 21 = 40%


So, Number of people that read Dainik Jagran = 40% of 2000 = 800
Hence, option (c) is the correct answer.

25 (d)
Given, Speed of the trains are 90 km/h and 108 km/h.
!
So, speed of the slower train = 90 km/h = 90× "# m/s = 25 m/s
!
And speed of the faster train = 108 km/h = 108× "# m/s = 30 m/s.
To completely cross each other, the trains have to effectively travel a distance equal to the sum of their lengths.
They cover this distance at an effective speed of (25 + 30) m/s in 10 seconds, i.e. 55 m/s in 10 seconds.
So, the sum of the lengths of the trains = 55×10 = 550 m.
For a passenger sitting in the faster train, to cross the slower train completely, he should have moved through a
distance equal to the length of the slower train.
Since the trains are moving in the same direction, the effective/relative speed = 30 – 25 = 5 m/s.
9 www.visionias.in ©Vision IAS

https://t.me/visioniastestseries2024
https://upscmaterial.online/

.
Since the distance equal to the length of the slower train is covered in 25 seconds, the length of the slower train =
5×25 = 125 m.
Thus, the length of the faster train = 550 – 125 = 425 m.
Hence, option (d) is the correct answer.

26 (b)
The given series is a combination of the following two series:
8, 9, 10, 11....
11, 12, 13, 14 ....
The next term is 11.
Hence, option (b) is the correct answer.

27 (c)
Brother of my wife→ my brother-in-law; Son of Asha’s brother is the brother-in-law of John. So, Asha’s brother
is John’s father-in-law, i.e., Asha is the sister of John’s father-in-law.

Hence, option (c) is the correct answer.

28 (d)
Statement 1 is not correct. The passage clearly mentions that a successful launch is only the first step towards a
long journey for the spacecraft. Therefore, we cannot say that a successful launch is the most important step.
Hence, it is not a correct statement.
Statement 2 is not correct. According to the author, India would be the fourth country to successfully land its
mission on moon. It is possible that there could be many other countries trying to land on moon who have had a
tough luck so far. So, we cannot conclusively say that only four countries have attempted to reach the moon so
far.

29 (c)
Statement 1 is correct. The passage says, “The French Revolution (1789-1799) not just changed France forever,
it had ripple effects across Europe and its colonies, shaping the concepts of democracy and equality as we know
them today”. It can be inferred that modern democracy has been influenced by French Revolution. Hence, it is a
correct statement.
Statement 2 is correct. When the king of France took an oath, the country was already plagued by economic
distress, which is attributed to corruption of the elite. The passage says, “When Marie Antoinette’s husband, Louis
XVI, inherited the throne in 1774, the treasury was already depleted, in a large part due to the decadence and
corruption of the Paris elite”. It can be inferred that corruption of elite society members may cause economic
distress. Hence, it is a correct statement.

10 www.visionias.in ©Vision IAS

https://t.me/visioniastestseries2024
https://upscmaterial.online/

.
30 (c)
Option (a) is not correct. In the passage, she says that if the subjects could not afford bread, they should switch
over to cake. The intent was not to make fun, but rather to suggest another food option. Hence, it is not a correct
option.
Option (b) is not correct. The passage nowhere mentions her generosity. Hence, it is not a correct option.
Option (c) is correct. The passage says, “In the 1780s, crops failed and prices rose to such an extent that even
bread became unaffordable for many. It is in this context that the Queen, when told her subjects could not afford
the staple food of bread, is believed to have said, then “let them eat cake”. The fact that the queen is unaware of
failed crops and price rise indicates that she is unaware of the world outside her palaces. Further, when she
suggests her subjects to have cake instead of bread, it reflects her lack of awareness regarding the cake being more
expensive than bread. Her ignorance is reflected in this line from the passage – “The massive folly of the sentence
is that the Queen apparently did not know that cake was more expensive than bread, and that people who couldn’t
afford the latter could most definitely not buy the former.” Hence, it is the correct option.
Option (d) is not correct: The passage nowhere mentions that the queen wanted her subjects to lead a good life.
Her suggestion regarding cake was out of her ignorance, rather than being a generous gesture, or reflective of her
will for them to have a good quality life. Hence, it is not a correct option.

31 (d)
It’s given that, P + Q, P x T, T + R, R ± S
Or P > Q, P ≥ T, T > R, R = S
Using this we get: S = R < T ≤ P > Q

Conclusion I: Q-T or Q ≤ T. This is not necessarily true.


Conclusion II: S±Q or S = Q. This is not necessarily true.

Thus, neither Conclusion-1 nor Conclusion-2 follows from the Statement.


Hence, option (d) is correct.

32 (c)
To find out which day it is exactly 1007 days from a given day, we need to calculate the remainder when dividing
1007 by 7, as the days of a week repeat in a cycle of 7.
Remainder [1007/7] = 6
So, if today is Friday, then 1007 days from today would be 6 days ahead in the week. Counting forward from
Friday, the day would be Thursday.
Hence, option (c) is correct.

33 (b)
Let the income of Parul and Palak be 4x and 3x respectively
And the Expenses of Parul and Palak are 3y and 2y respectively.
It is given that each of them saves Rs. 7000 monthly.
So, Savings of Parul = 4x – 3y = 7000........................(1)
Savings of Palak = 3x -2y = 7000..........................(2)

Subtracting (2) from (1), we get:


x –y = 0, i.e. x = y..................................(3)
Using (3) in (1) we get,
x = 7000.
11 www.visionias.in ©Vision IAS

https://t.me/visioniastestseries2024
https://upscmaterial.online/

.
So, Parul’s Income = 4x = 28000
Palak’s income = 3x = 21000
Net difference = 28000 – 21000 = 7000
Required percentage = (7000/21000) × 100% = 33.33%
Hence, option (b) is the correct answer.

34 (d)
Here, we are required to decipher or decode the pattern.
PENCIL is written as QGQGNR.
The underlying pattern is shown below:
P+1=Q
E+2=G
N+3=Q
C+4=G
I+5=N
L+6=R

We will follow a similar pattern to code ERASER.


E+1=F
R+2=T
A+3=D
S+4=W
E+5=J
R+6=X
So, the required code is FTDWJX.
Hence, option (d) is correct.

35 (d)
We can break the given sequence in five sets of toplap. Letters of toplap are alternatively in large and small caps.
The complete sequence is: ToPlAp/ToPlAp/ToPlAp/ToPlAp/ToPlAp
Hence, option (d) is correct.

36 (d)
Statement 1 is correct. It is mentioned in the first line of the passage, “Foreign portfolio investors (FPIs) are
driving up the stock markets to new peaks on a daily basis, sending investors into a buying euphoria. After
renewed interest from FPIs helped the benchmark Sensex surge by around 10 per cent in the first quarter of fiscal
2024”. Hence, it is a correct statement.
Statement 2 is correct. It is mentioned in the passage, “The major driver is the return of FPIs, the buoyancy in
the global markets, strong macroeconomic fundamentals and the easing of inflation in India”. This implies that
among a number of factors for the surge in Stock markets, the decline in inflation in India is also a reason for the
same. Hence, it is a correct statement.
Statement 3 is not correct. The passage says, “US Fed is near the end of the rate hiking cycle”, but it does not
say that the rate hike has ceased. Hence, it is not a correct statement.

12 www.visionias.in ©Vision IAS

https://t.me/visioniastestseries2024
https://upscmaterial.online/

.
Statement 4 is correct: The passage says, “Consequently, the US 10-year bond yield has sharply dipped from
5.1% to 4.7% and the Dollar Index has crashed by nearly 4% from 103.57 to 99.9. This is positive for emerging
markets like India, which are likely to witness more capital flows”. Hence, the crash of the dollar index is a
positive indication for developing countries like India, as it will lead to more influx of capital. Hence, it is a
correct statement.

37 (a)
Statement 1 is correct. The endurance of the US economy is a major support for the markets worldwide. The
passage says, “…this resilience of the US economy, which was not anticipated and discounted by the market, is
the strongest pillar of support for the global markets now, he said”. Hence, it is a correct statement.
Statement 2 is not correct. The passage mentions that there has been a surge in the inflow of capital in the Indian
stock market. However, the passage does not claim that India has become the fastest-growing major economy
owing to the surge in FPI. Hence, it is not a correct statement.

38 (a)
Option (a) is correct. What the passage means/implies is reflected in the following line - “…a great deal of
corruption, which affects the ordinary person, is at the retail level.”
Option (b) is incorrect. The given option may be considered partially correct because of the phrase politicization
of institutions, even though it may be considered bit of a stretch. But the context of lax attitude of citizens is not
correct, as it is not mentioned in the passage. So, this option is not what the passage implies.
Option (c) is incorrect. This option is limited to only the anti-corruption institutions and their reform. It’s very
generic in nature. Moreover, the context of the impact of corruption on ordinary man is missing, which has been
covered in option (a). Hence, this is not what the passage implies.
Option (d) is incorrect. This option is very generic and broad and does not cover the specific context of the
passage. Also, the issue of citizen awareness has not been covered in the passage. Hence, this option is not what
the passage implies.

39 (c)
Option (a) is incorrect. The given option is very generic, broad and idealistic. Also, it is not based on the
information given in the passage, as the context of pledging for improving the conditions of women is not a part of
the passage. So, this option is not the best crux of the passage.
Option (b) is incorrect. The passage is focused on issues which women in society face – patriarchal mindset is an
integral component. Therefore, it would be grossly incorrect to say that women’s empowerment is possible even if
patriarchal mindset continues. Hence, this answer option is incorrect.
Option (c) is correct. The lines “The conventional thinking is that … These thinkers are entirely incapable of
grasping the horrors of a world without women. If girls are born, they do not receive the same commitment and
care as boys would”, show that the present status of women is due to the patriarchal mindset and the gender
inequality rampant in rural society. They are among the reasons blocking women empowerment.
Option (d) is incorrect. The given option could be a rational inference, even though the context of the
government’s role is not a part of the passage, but it is not the best crux.

40 (b)
Assumption 1 is incorrect. Refer to the lines “Although it has never had a standardized diet, India has
traditionally “imagined” its cuisine concerning the incorporation and domestication of “foreign” influences. … a
variety of multinational fast-food companies have entered the previously protected Indian culinary landscape. …
These companies have had to “Indianize” and self-domesticate to conquer the notoriously difficult-to-please
Indian palate.” There is an influx of multi-national companies. However, there never was a “standard” Indian
diet, and there is no compromise because of such companies. Hence, this assumption is not correct.

13 www.visionias.in ©Vision IAS

https://t.me/visioniastestseries2024
https://upscmaterial.online/

.
Assumption 2 is correct. The following lines “The Indian food market of $182 billion is believed to be growing
at a rapid clip of 13 per cent. Indian precooked packaged foods empires such as MTR, SWAD, Haldirams, and
Pataks have gone global, available wherever Indians now live, leading a quiet yet unrecognized revolution in
eating habits”, mean that demand for Indian food is still there among Indians living abroad. Hence, it confirms the
validity of the given assumption.

41 (c)
It is given that,
b = (2/3) c
c = (2/3) d
⇒ b = (2/3) × (2/3) d = (4/9) d
Hence, option (c) is correct.

42 (d)
The respective positions of the towns have been represented below:

Clearly, town K is in the South-East direction from town D.


Hence, option (d) is correct.

43 (d)
$%&'( *%*+('&,%- %. &/0 1&'&0 "
Value of the vote of an elected MLA = ( $%&'( -+2304 %. 0(05&06 2023041 ,- &/0 1&'&0 (07,1('&,80 '11023(9 ) × "::: ,

For Uttar Pradesh, Value of the vote of an elected MLA = 208 and the total number of elected members in the
state legislative assembly = 403.
$%&'( *%*+('&,%- %. &/0 1&'&0 "
So, 208 = ( ;:<
) × ":::

So, total population of the state (Uttar Pradesh) = 208×403×1000 = 83824000 = 8.38 Crore (approximately)
Hence, option (d) is the correct answer.

14 www.visionias.in ©Vision IAS

https://t.me/visioniastestseries2024
https://upscmaterial.online/

.
44 (c)
$%&'( 8'(+0 %. 8%&01 %. '(( =>?' %. '(( 1&'&01
Value of vote of an elected MP = $%&'( -+2304 %. 0(05&06 2023041 %. @'4(,'20-&
Given that: The vote value of each elected MP = 708 and the total value of votes of all MLAs of all states is
549408.
Also, given that the number of elected members in the Lok Sabha = 543.
Let the number of elected members in the Rajya Sabha = x.
!;A;:#
So, 708 =
!;<BC
!;A;:#
⇒ 543 + x =
D:#

⇒ 543 + x = 776
⇒ x = 776 – 543
∴ x = 233
Hence, option (c) is the correct answer.

45 (c)
The various possible arrangements are:
i) 2 boys from amongst Pankaj’s friends and 2 girls from amongst Suman’s friends.
Required no. of ways = 2C2×2C2
OR
ii) 1 boy and 1 girl from amongst Pankaj’s friends and 1 boy and 1 girl from amongst Suman’s friends.
Required no. of ways = 3C1 × 2C1 × 3C1 × 2C1
OR
iii) 2 boys from amongst Suman’s friends and 2 girls from amongst Pankaj’s friends.
Required no. of ways = 3C2×3C2

Total number of ways = 2C2×2C2 + 3C1 × 2C1 × 3C1 × 2C1 + 3C2×3C2 = 1 + 36 + 9 = 46


Hence, option (c) is the correct answer.

46 (b)
Number of digits used from pages 89 to 99 = 11 × 2 = 22
Number of digits used from pages 100 to 143 = 44 × 3 = 132
Total number of digits used from pages 89 to 143 = 22 + 132 = 154
Hence, option (b) is the correct answer.

47 (c)
Options (a) and (b) are wrong as it’s not necessary that the concerned person goes to Goa or tries to be with family
members whenever he needs leisure. He may be having other options too.
The conversion of the first statement gives us the answer, “If I do not need leisure, I do not go to Goa” as I go to
Goa only if I need leisure. Hence, option (c) is the correct answer.

15 www.visionias.in ©Vision IAS

https://t.me/visioniastestseries2024
https://upscmaterial.online/

.
48 (b)
Substance A:
" <
Sum of ratios of ingredients X and Y = ; + ; = 1
So, the quantity of the possible ingredient Z = 0
(Sum of ratios of all the ingredients in a substance must be 1)
Quantity of ingredient X in 2 kg of the substance (A) = 2/4 = 1/2 kg,
"
and the quantity of ingredient Y in 2 kg of the substance (A) = 2 − E = 3/2 kg.

Substance B:
" !
Sum of ratios of ingredient X and Y = + = 1
F F

So, the quantity of the possible ingredient Z = 0


Quantity of ingredient X in 3 kg of the substance (B) = 3/6 = 1/2 kg,
"
and the quantity of ingredient Y in 3 kg of the substance (B) = 3− E = 5/2 kg.

" "
Therefore, the quantity of ingredient X in substance C (the mixture) = + = 1 kg,
E E
< !
and the quantity of ingredient Y in the substance C = + = 4 kg.
E E
"
Hence, the required ratio = = 1 : 4.
;

Hence, option (b) is the correct answer.

49 (a)
Let number of questions attempted wrong by Ram = y
Then, number of questions attempted right by Ram = (6 - y)
Hence, (6 - y) - y/4 = 2.25
or {4(6 - y) - y}/4 = 2.25
or (24 - 4y - y)/4 = 2.25
or 24 - 5y = 2.25 × 4
or 24 - 5y = 9
or 5y = 24 - 9
or 5y = 15
or y = 3

Similarly, let number of questions attempted wrong by Shyam = x


Then, number of questions attempted right by Shyam = (7 - x)
Hence, (7 - x) - x/4 = 5.75
or {4(7 - x) - x}/4 = 5.75
or (28 - 4x - x)/4 = 5.75
or 28 - 5x = 5.75 × 4

16 www.visionias.in ©Vision IAS

https://t.me/visioniastestseries2024
https://upscmaterial.online/

.
or 28 - 5x = 23
or 5x = 5
or x = 1
Hence (3, 1) is the correct answer.
Hence, option (a) is the correct answer.

50 (b)
Option (a) is incorrect. The passage is primarily about work productivity. The author mentions how more than
health, it is the desire to work that matters for productivity. The health aspect is a peripheral argument that lends
credence to the core theme of the passage. Moreover, this statement may be a practical implication, but is not the
crux of the passage.
Option (b) is correct. The line “Productivity is largely a factor of an employee's desire to work. Just because they
have the physical capacity to do more work doesn't necessarily mean they will.” from the passage reflects the best
crux of the passage. These lines show that for productivity of an organization the employee should be motivated
(desire to work). Though a good health could increase the ability to work. This answer option covers both these
aspects. Hence, this option is the best crux of the passage.
Option (c) is incorrect. The given option is not correct because the productivity of an employee is not directly
related to his health. The lines “Productivity is largely a factor of an employee's desire to work. Just because they
have the physical capacity to do more work doesn't necessarily mean they will.” reflect that productivity depends
on the desire to work and not just on the health of the employee. Hence, this is not the best crux of the passage.
Option (d) is incorrect. The given option could be a practical implication, but not the crux. Therefore, this
answer option is incorrect.

51 (d)
Option (a) is incorrect. The given option is not correct because, in the context of this passage, urban areas face
the challenges posed by climate change, and not urbanization per se. The passage does not mention any challenges
posed by urbanization. It mentions how urbanization exacerbates climate change. Moreover, the author is silent on
the topic of planned or unplanned urbanization. Therefore, the given option is not the best crux of the passage.
Option (b) is incorrect. The context of rural areas is completely missing from the information covered in the
passage. Hence, this option is beyond the scope of the passage and is not correct.
Option (c) is incorrect. The passage does not mention anything about “controlled” urbanization. Also, the given
option is a kind of rational implication, as the effects of urbanization on climate change can be controlled through
planned or controlled urbanization. Therefore, the given option is not the best crux of the passage.
Option (d) is correct. The crux as stated in the option is seen in the lines “At the same time, cities are a key
contributor to climate change, as urban activities are major sources of greenhouse gas emissions. Estimates
suggest that cities are responsible for 75 per cent of global CO2 emissions, with transport and buildings being
among the largest contributors.” Also, “Climate change is a global phenomenon that largely impacts urban life.
… All these have costly impacts on cities' basic services, infrastructure, housing, human livelihoods, and health.”
These lines validate that urbanization exacerbates the impacts of climate change. Hence, this is the best crux of the
passage.

52 (d)
Inference 1 is incorrect. The given inference is not correct because of the lines “Though investment in EdTech
has been increasing, learning and outcomes, as a result, have not changed considerably in many countries. An
OECD report found that when it comes to the impact of computer usage in schools as measured through PISA,
“impact on student performance is mixed, at best."” Despite the increase in investments and enhanced usage of
technology, it is probable that educational outcomes might not improve. So, this inference is not correct.

17 www.visionias.in ©Vision IAS

https://t.me/visioniastestseries2024
https://upscmaterial.online/

.
Inference 2 is incorrect. The context of emotional intelligence of teachers is not a part of the passage. Therefore,
this inference is beyond the scope of the passage and is not correct.

53 (b)
Option (a) is incorrect. The author recommends that we should focus on the social aspects of technology to
enhance connections from a distance. However, the passage nowhere talks about inclusive growth of the students
because of this as such.
Option (b) is correct. The given option is not correct because as per the passage “Much more attention must be
directed to how technology will enhance teaching and learning in a blended learning environment reaching
students, both in school and at home.” Technology may help students in both environments, and is not limited to
remote areas only.
Option (c) is incorrect. The lines “While we can never replace the magic that happens between great teachers
and students in an in-person environment, we should focus on the social aspects of technology to enhance
connections from a distance”, clearly explain that it is not desirable to replace physical teaching with online
teaching. It also goes against the tone and flavour of the passage. Hence, the given statement is not correct.
Option (d) is incorrect. The following lines from the passage “… we should focus on the social aspects of
technology to enhance connections from a distance. Much more attention must be directed to how technology will
enhance teaching and learning in a blended learning environment reaching students, both in school and at
home”, show that only using or access to technology is not enough; we need more attention on enhancing teaching
and learning in a mixed environment. However, it does not mean that teaching quality cannot improve by giving
teachers access to technology. Technology may be a necessary but not a sufficient condition. Hence, the given
statement is not correct.

54 (d)
The question is about finding the invalid assumptions (not the valid ones).
Assumption 1 is correct. The lines “It has been noted that the younger generation falls prey to such cybercrimes
the most. This is majorly because of their immaturity, which is easily identified by these criminal minds”, validate
the assumption given in the option. Hence, the given assumption is correct as per the passage.
Assumption 2 is correct. The given assumption is correct as per the lines “Tons of cyber information available
online has opened the gates for new legal challenges for which adequate laws are yet to be framed.” The
required laws are yet to be framed. That clearly means the existing laws are still evolving. So, as per the passage,
the given assumption is correct.

55 (c)
Option (a) is incorrect. The context of upgrading the cybersecurity software, protocols and algorithms for
controlling cybercrimes is not covered in the passage. Therefore, the given option is beyond the scope of the
passage and is not correct.
Option (b) is incorrect. The line “Social media is a form of communication via the Internet. Its main goal when it
came into being, was to create a virtual kinship network throughout the world” states that social media is a form
of communication via the Internet. However, whether or not it is the prime medium behind the misuse of personal
information has not been stated or alluded to by the author. Hence, this option is not correct.
Option (c) is correct. The lines “Tons of cyber information available online has opened the gates for new legal
challenges for which adequate laws are yet to be framed” and “This is major because of their immaturity, which
is easily identified by these criminal minds”, reflect that along with improving knowledge of youth we need laws
for preventing misuse of the information shared online. Hence, this inference is correct.
Option (d) is incorrect. The context of preparing cyber security apparatus for future challenges is not the best
inference as this is not discussed in the passage. Hence, this option is not correct.

18 www.visionias.in ©Vision IAS

https://t.me/visioniastestseries2024
https://upscmaterial.online/

.
56 (a)
Let S be main station, and s1, s2, s3, s4, s5 and s6 be the 6 substations.

Distance travelled in installing 6th substation = 2 × (5 + 6 + 7 + 8 + 9 + 10) = 2 × 45 = 90 jumps


Distance travelled in installing 5th substation = 2 × (5 + 6 + 7 + 8 + 9) = 2 × 35 = 70 jumps
Distance travelled in installing 4th substation = 2 × (5 + 6 + 7 + 8) = 2 × 26 = 52 jumps
Distance travelled in installing 3rd substation = 2 × (5 + 6 + 7) = 2 × 18 = 36 jumps
Distance travelled in installing 2nd substation = 2 × (5 + 6) = 2 × 11 = 22 jumps
Distance travelled in installing 1st substation = 2 × 5 = 10 jumps

One jump of a man on the Moon is equal to 5 meters.


Total distance travelled in installing 6 substations = 5 × {90 + 70 + 52 + 36 + 22 + 10} = 5 × 280 = 1400 meters
Hence, option (a) is the correct answer.

57 (b)
Let the number of opponents in the first round be x.
So, the number of supporters = 500 - x

In the second round, the number of opponents = 2x


And the number of supporters = 500 - 2x

According to the question,


500 - x = 3(500 - 2x)
Or 1500 - 6x = 500 - x
Or 1000 = 5x
Or x = 1000/5 = 200
Hence, option (b) is correct.

58 (a)
There are two conditions:
i. If there are x number of soldiers then the maximum number of hand grenades that a soldier may carry = x
ii. No soldier may carry 55 hand grenades.
If there is only one soldier, then he will carry only 1 hand grenade.
If there are two soldiers, then they will carry 1 and 2 number of hand grenades.
If there are three soldiers, then they will carry 1, 2 and 3 number of hand grenades and so on.
Now, if there are 54 soldiers, then they will carry 1, 2,…54 number of hand grenades.
But if there are 55 soldiers, then 54 soldiers may be carrying different number of hand grenades. However, as no
one can carry 55 hand grenades, one soldier is left unaccounted for.
19 www.visionias.in ©Vision IAS

https://t.me/visioniastestseries2024
https://upscmaterial.online/

.
Similar will be the case for higher number of soldiers. Hence, maximum possible number of soldiers in that
particular platoon = 54.
Hence, option (a) is correct.

59 (a)
Time from 12 P.M. on Friday to 2 P.M. on the following Friday = 7 days + 2 hours = 170 hours.
Time that the watch gained in 170 hours = 2 + 4 + (48/60) = 34/5 minutes
Now, the watch gains (34/5) minutes in 170 hours
∴ The watch gains 1 minute in 170 / (34/5) hours
∴ The watch gains 2 minutes in (170 × 5 × 2)/ 34 = 50 hours
Hence the watch will show correct time after 50 hours or 2 days and 2 hours.
Hence, the watch was correct on Friday + 2 days and 2 hours = Sunday 2 p.m.
Hence, he reached Gwalior at 2 P.M. on Sunday.
Hence, option (a) is correct.

60 (b)
We have to find the number of internal cubes, i.e. the number of cubes that are not exposed.
There are 343 cubes.
Now, 73 = 343
So, n = 7
Number of internal cubes = (n – 2)3 = (7 – 2)3 = 53 = 125
Hence, option (b) is correct.

61 (d)
On counting from top it is found that, the second, fifth and sixth books are of Tamil. So, we get:
1.
2. Tamil
3.
4.
5. Tamil
6. Tamil
7.
8.

One book of Tamil is just between the two books of Bengali. So, we get:
1. Bengali
2. Tamil
3. Bengali
4.
5. Tamil
20 www.visionias.in ©Vision IAS

https://t.me/visioniastestseries2024
https://upscmaterial.online/

.
6. Tamil
7.
8.

Two books of Tamil are between the two books of Marathi. So, we get:
1. Bengali
2. Tamil
3. Bengali
4. Marathi
5. Tamil
6. Tamil
7. Marathi
8.

Hindi book is below the Marathi books. So, on arranging the books according to the given conditions, we get the
following arrangement:
1. Bengali
2. Tamil
3. Bengali
4. Marathi
5. Tamil
6. Tamil
7. Marathi
8. Hindi

Marathi book is fifth from below.


Hence, option (d) is the correct answer.

62 (b)
Option (a) is incorrect. The passage does not mention any other profession which has included women’s
perception. Hence, this option is beyond the scope of the passage.
Option (b) is correct. Refer to the lines “When judges interpret and implement the law, their reasons and
opinions reflect their thought process, an insight into their perceptions. These perceptions at the very least must
be representative of both men and women on the bench to ensure a fair and adequate response through judicial
decisions”. These lines show that it is important to give adequate importance to women point of view, as also
reflected in the line “It is about integrating the gender perspective and giving equal visibility to women.”
Option (c) is incorrect. The context of delayed justice for women and lack of representation being its cause is not
described in the passage. Hence, this option is not what the passage implies.
Option (d) is incorrect. The given option is not correct because of the lines “It is important to note that including
women in the judiciary is not simply about ensuring that her perception is relevant to resolving cases about
women. It is much more than that. It is about integrating the gender perspective and giving equal visibility to
women.” Hence, the given option is not correct.

21 www.visionias.in ©Vision IAS

https://t.me/visioniastestseries2024
https://upscmaterial.online/

.
63 (c)
Option (a) is incorrect. The context of pressurizing children intentionally or unintentionally is not discussed in
the passage. The lines “Be a winner, be a topper. That is what everybody wants the school, teachers, parents, or
even our relatives”, show that probably parents and teachers intentionally desire winners, creating pressure on the
children. Even if true, this statement cannot be the crux of the passage.
Option (b) is incorrect. This is a generic statement which is not based on the information given in the passage.
Hence, this option is beyond the scope of the passage.
Option (c) is correct. Refer to the lines “Every parent wants their child to be successful in life. Be a winner, be a
topper. That is what everybody wants the school, teachers, parents, or even our relatives (author highlighting
unfair expectations). … But sometimes, they must remember to evaluate whether children are motivated (intrinsic
drive ) or stressed.” This summarizes the core theme of the passage which is reflected in the given option. Hence,
this option is the best crux of the passage.
Option (d) is incorrect. Although this option may reflect a rational inference, it is not the crux of the passage.
The crux is the essence of the passage which is reflected in option (c). Also, this option is limited to parents, while
the author talks about academic pressure in general – “Academic pressure causes students to feel more stressed
and anxious, negatively affecting their physical, social, and emotional well-being.” Moreover, the author does not
say or allude to the fact that students’ happiness is a bigger virtue than their pursuit for academic excellence. The
tone of the passage is pretty balanced, which indicates that one should not be sacrificed for the sake of the other.

64 (b)
Assumption 1 is incorrect. The lines “The marine fish farming segment is hitherto untapped and establishing
commercial hatchery technologies and suitable farming methods is yet another significant opportunity, given the
country’s vast coastline”, show that commercial hatchery technologies are related to marine fish farming and not
freshwater. At least the passage does not mention commercial hatchery technologies in relation with freshwater
fishing. Even if true, whether it could make India the number one in freshwater fish-farmed production cannot be
deduced from the information provided. So, this assumption is not correct.
Assumption 2 is correct. The lines “A large extent of Indian aquaculture is still based on traditional farming
methods. Converting them to modern farming methods will increase fish production and will also address the
sustainability of the environment.” reflect that assuming modern farming methods to be more environmentally
friendly would be correct.

65 (a)
Option (a) is correct. This option is based on the following lines “There is immense scope for its development
when improved systems and species are adopted” and “The marine fish farming segment is hitherto untapped and
establishing commercial hatchery technologies and suitable farming methods is yet another significant
opportunity, given the country’s vast coastline.” Therefore, this option best captures the essence of the passage.
Option (b) is incorrect. “Marine fishing is more beneficial than freshwater fish farming” cannot be claimed based
on the information given in the passage. Hence, this is not the best crux of the passage.
Option (c) is incorrect. The relationship between India’s marine environment and it being a limitation for India in
becoming the leader in fish production is not discussed in the passage. The line “India holds the second position in
the world for freshwater fish-farmed production” is specific to freshwater and not marine water. Hence, this
option is beyond the scope of the passage.
Option (d) is incorrect. The given option is not correct because the environmental impact of any change in the
technology of fishing is not the main theme of the passage.

22 www.visionias.in ©Vision IAS

https://t.me/visioniastestseries2024
https://upscmaterial.online/

.
66 (c)
p - 2/3(2p - 3q) - 1/3(p + 4q)
= p – (4/3)p + 2q – (1/3)p – (4/3)q
= (-2/3) p + (2/3)q
All the answer choices except (c) are equivalent to (-2/3) p + (2/3)q
Hence, option (c) is the correct answer.

67 (a)
Considering series of the inner circle –
0, 1, y, 2, 3, 5, 8, 13
The above series is a Fibonacci sequence. In Fibonacci sequence each number is the sum of the two preceding
ones, starting from 0 and 1.

Hence, y = 0 + 1 = 1

Considering series of the outer circle:


0, 8, 8, 16, 24, 40, 64, x
Here, each term is the sum of two preceding ones.
Hence, x = 40 + 64 = 104
Now, x + y = 104 + 1 = 105
Hence, option (a) is the correct answer.

68 (a)
Let the marked price be Rs. 100.
Regular discount = 15%,
Price after first discount = Rs. 85

Case (i): When festive season discount = 10%


Selling price = 85 × 0.9 = Rs. 76.5.
On selling at Rs. 76.5 there is a profit of 8.49%
So, Cost price = [76.5/(100 + 8.49)] × 100 = Rs. 70.5

Case (ii): When festive season discount = 7%


Selling price = 85 × 0.93 = Rs. 79.05
So, required profit percentage = [(79.05 – 70.5)/70.5] × 100 = 12.12%
Hence, option (a) is the correct answer.

23 www.visionias.in ©Vision IAS

https://t.me/visioniastestseries2024
https://upscmaterial.online/

.
Explanation for Questions 69 and 70:
Since, each of the five teams played exactly one match with other teams; therefore, every team played 4 matches
each. Now, tabulating the given information.
Putting figures as per the given statements (i) and (ii):

Team Number of matches won Number of matches lost

A 1 or 3 3 or 1

D 3 or 1 1 or 3

E 0 4

However, as given in statement (iii), the number of matches won by D was more than that by B. Hence, D cannot
win just 1 match. So, it must have won 3 matches.

Team Number of matches won Number of matches lost

A 1 3

D 3 1

E 0 4

Now again as per statement (iii), the number of matches won by D is more than that of B, and no team won the
same number of matches. So, the final picture that emerges has been tabulated below:

Team Number of matches won Number of matches lost

A 1 3

B 2 2

C 4 0

D 3 1

E 0 4

69 (b)
Total number of matches lost by B, C and D together = 2 + 0 + 1 = 3
Hence, option (b) is the correct answer.

24 www.visionias.in ©Vision IAS

https://t.me/visioniastestseries2024
https://upscmaterial.online/

.
70 (c)
From the above table it is clear that team C has won the maximum number of matches.
Hence, option (c) is the correct answer.

Explanation for Questions 71 to 73:


Using information from statements 1, 6, and 10, we get:

Station Gets On Gets Down

Base Station ___ xxx

1 xxx

2 xxx

5 xxx

Using information from statements 4, 5 and 9, we get:

)
om
Station Gets On Gets Down

l.c
ai
gm
Base Station ___ xxx

5@
02
1 xxx
24
91

2 xxx
gs
h(
ng

3 G only B, D
Si
ep
de

4 A only
ur
rG

5 xxx A, G, C
fo
ed
is

As per statement 8, E gets on with two others and gets down alone at a station that is subsequent from the one at
al
on

which D gets down. So, E got down at station 4.


rs
pe

As per statement 2, D gets down at the next station from the one at which F gets down. So, F got down at station
is
t

2. So, we get:
en
m
cu

Station Gets On Gets Down


do
is
Th

Base Station ___ xxx

1 xxx

2 xxx F

3 G only B, D

4 A only E

5 xxx A, G, C

25 www.visionias.in ©Vision IAS

https://t.me/visioniastestseries2024
https://upscmaterial.online/

.
Using statement 7, we know that C gets on with F but does not get on with either B or D. So, if C and F board the
train at the base station (along with E), B and D must board the train at station 1. And if C and F board the train at
station 1, B and D must board the train at the base station (along with E). So, we get:

Station Gets On Gets Down

Base Station E and (C, F) or (B, D) xxx

1 (C, F) or (B, D) xxx

2 xxx F

3 G only B, D

4 A only E

5 xxx A, G, C

71 (d)
72 (c)
73 (d)

74 (a)
3 * 4 $ 5 = [{3 × 4 + 3 × 5} × (5 - 3)] = 27 × 2 = 54

Similarly,
2 * 3 $ 5 = [{2 × 3 + 2 × 5} × (5 - 2)] = 16 × 3 = 48

Similarly,
5 * 6 $ 7 = [{5 × 6 + 5 × 7} × (7 - 5)] = 65 × 2 = 130
Hence, option (a) is the correct answer.

75 (b)
Let after x months Rakesh joined Anuj in business.
Rakesh invested 33.33% more amount than Anuj.
33.33% = 1/3
Ratio of amounts of Anuj and Rakesh is 3 : 4.
Ratio of time period of investment of Anuj and Rakesh is 12 : (12 - x).
So, Ratio of time-amount value of Anuj and Rakesh = 3 × 12 : 4 × (12 - x) = 9 : (12 - x)

Total profit = Rs.17408


Anuj’s profit = Rs.9216
So, Rakesh’s profit = 17408 - 9216 = Rs.8192
So, 9/(12 - x) = 9216/8192

26 www.visionias.in ©Vision IAS

https://t.me/visioniastestseries2024
https://upscmaterial.online/

.
Or 9/(12 - x) = 9/8
Or 12 – x = 8
Or x = 4 months

Hence, ratio of time period of investment of Anuj and Rakesh = 12 : (12 - 4) or 3 : 2


Hence, option (b) is the correct answer.

76 (b)
Option (a) is incorrect. The passage is not about the measurement of the value of stolen artefacts, that too based
on their historical importance. Therefore, this option is beyond the scope of the passage and is not correct.
Option (b) is correct. Refer to the lines “The illegal trade in smuggled antiques is a profitable venture for
smugglers. It has become a global concern today” and “The country's struggles for conserving and recovering its
artefacts have seen considerable success so far. Nevertheless, there is still a long journey ahead”. These lines
illustrate that the given option best captures the essence of the passage.
Option (c) is incorrect. The context of global consensus and information sharing concerning artefacts is not
covered in the passage. Hence, this option is not the correct crux.
Option (d) is incorrect. The passage mentions a law in the line - “To combat this problem, India passed the
Antiquities and Art Treasures Act in 1972 for the protection and preservation of valuable objects of the past.”, but
it does not state that the implementation of the law is poor. Hence, this option is not the correct crux of the
passage.

77 (d)
Assumption 1 is incorrect. The given assumption is not correct because the passage does not compare the role of
local and national governments in the process of climate adaptation. It only mentions that local governments need
to play a pioneering role in adapting to climate change as per the lines “Local governments are therefore at the
frontline of adaptation.”
Assumption 2 is incorrect. Refer to the lines, “throughout history, people and societies have adjusted to and
coped with changes in climate and extremes with varying degrees of success.”. Here, the author talks about how
people and societies have adjusted and coped with changes in climate. There is however, no mention of their role
in accelerating climate change. Hence, this assumption is not correct.

78 (a)
Option (a) is correct. The given option is based on the following lines “Our modern life is tailored to that stable
climate, not the much warmer climate of the next thousand years. As our climate changes, we will need to adapt”
and “Climate change (drought in particular) has been at least partly responsible for the rise and fall of
civilizations”. These lines show that climate change causes devastation, but if we need to live our modern lives
then we need to adapt to the changing climate. Further the lines, “Local governments are therefore at the frontline
of adaptation. Cities and local communities around the world have been focusing on solving their climate
problems.” emphasise the role of local-level adaptation measures.
Option (b) is incorrect. The given option is not correct because it mentions ‘reversal’ of climate change with
intervention at local levels. However, the passage focuses on adaptation to and not reversal of climate change at
the local level as is clear from the lines, “Local governments are therefore at the frontline of adaptation. Cities
and local communities around the world have been focusing on solving their climate problems.” Hence, as per the
passage, this is not the best crux of the passage.
Option (c) is incorrect. The option adds a new context of ‘climate change mitigation’, which is not a part of the
passage. The passage only focuses on the adaptation to climate change as per the lines “Local governments are
therefore at the frontline of adaptation. Cities and local communities around the world have been focusing on
solving their climate problems.” So, this option does not reflect the crux of the passage.

27 www.visionias.in ©Vision IAS

https://t.me/visioniastestseries2024
https://upscmaterial.online/

.
Option (d) is incorrect. The given option establishes a necessary relationship between adaptation measures at the
local level to solving climate change at the global level. This statement is logically correct; however, such a
relationship is not reflected in the passage. The passage states that since climate change is felt on a local level,
local governments are at the forefront of adaptation as per the line “The faster the climate changes, the more
difficult it will be. While climate change is a global issue, it is felt on a local scale. Local governments are
therefore at the frontline of adaptation.” However, to state that unless changes are done at the local level, climate
change cannot be addressed at the global level would not be correct.

79 (c)
Case l:

By looking at the diagram of case l we can say that conclusion l cannot be drawn because some Clerks are
Officers and conclusion III cannot be drawn because all Officers are not Graduates.

Case II:

By looking at the diagram of case II we can say that conclusion II can be drawn because all Clerks can be
Scientists. It is a possibility.
Hence, option (c) is the correct answer.

80 (b)
Number of persons who got flat number which is divisible by 6 = 16 persons (all multiples of 6 till 100)
Number of persons who got flat number which is divisible by 8 = 12 persons (all multiples of 8 till 100)
Number of persons who got flat number which is divisible by 8 & 6 both = 4 persons (L.C.M. of 8 & 6 = 24 i.e. all
multiples of 24 till 100)

So, the number of persons who got flat number which is divisible by 8 but not by 6 = 12 – (number of flats whose
number is divisible by 8 & 6 both)
= 12 – 4 = 8 persons
Number of persons who got flat number which is divisible by 8 but not by 6 got 5% discount.
Hence, 8 persons got 5% discount.
Hence, option (b) is the correct answer.

Copyright © by Vision IAS


All rights are reserved. No part of this document may be reproduced, stored in a retrieval system or
transmitted in any form or by any means, electronic, mechanical, photocopying, recording or otherwise,
without prior permission of Vision IAS.
28 www.visionias.in ©Vision IAS

https://t.me/visioniastestseries2024

You might also like